Free NCLEX Exam Questions

This free NCLEX Practice Exam 1 has 207 Questions. The questions include answers and detailed explanations. The NCLEX exam subjects include cardiovascular health problems, respiratory, upper gastrointestinal, lower gastrointestinal, digestion including gallbladder, liver and pancreas, endocrine, musculoskeletal, maternal, mental health, adolescent care, renal, adolescent care, fundamentals, priorities of care, health promotion and maintenance, safe and effective care, basic care and comfort, safety and infection control, reduction of risk, and management of care.

NCLEX Question 1: A 65-year-old patient with a history of coronary artery disease (CAD) is prescribed aspirin as part of their daily medication regimen. The nurse should instruct the patient that the primary purpose of aspirin in CAD management is to:
A) Lower blood pressure. 
B) Increase HDL cholesterol levels.
C) Inhibit platelet aggregation.
D) Reduce LDL cholesterol levels.

QBankproAcademy.com

Explanation: Aspirin is commonly prescribed in CAD management because it helps inhibit platelet aggregation, thereby reducing the risk of blood clots and thrombosis, which can lead to heart attacks and strokes. It is not primarily used to lower blood pressure (option A), increase HDL cholesterol levels (option B), or reduce LDL cholesterol levels (option D).

@qbankproacademy

4000+ Free NCLEX QUESTIONS. Go to QBankProAcademy.com FREE Qbank questions for NCLEX RN, PN, HESI Exit, Med Surg, AANP, ANCC, and HesiA2. Practice questions, quizzes, and listen to the Free Podcast. In this video, we review a question on CARDDIOGENIC SHOCK nursing care. At QbankproAcademy.com our mission is to provide free QBanks, videos, and the most up to date test prep information for nurses. If you find our website helpful, please tell other aspiring nurses, nursing students, and professors. Please link to our site from your blogs, videos, and college websites, or share us on your favorite social media sites. Thank you for your support! Free nursing NCLEX 60-DAY CHALLENGE https://qbankpro.thinkific.com/courses/qbankpro-academy #nclexquestions #nursehumor #nursingquestions #aanpquestion qbank #ancc #nursingtestbank qbank

♬ original sound – QBank Pro Academy – QBank Pro Academy

NCLEX Question 2: A patient with heart failure presents with sudden shortness of breath, wheezing, and a productive cough with pink frothy sputum. Which medication should the nurse anticipate administering as the initial treatment for this acute exacerbation of heart failure?
A) Furosemide (Lasix)
B) Lisinopril (Zestril)
C) Amlodipine (Norvasc)
D) Metoprolol (Lopressor)

QBankproAcademy.com

Explanation: In an acute exacerbation of heart failure with symptoms of pulmonary congestion, the nurse should anticipate administering a loop diuretic such as furosemide (Lasix) to reduce fluid overload and relieve symptoms. Lisinopril (option B), Amlodipine (option C), and Metoprolol (option D) are not the initial medications of choice for this situation.

“Our mission is to continue to provide free QBank, videos, and the most up to date test prep information for nurses. If you find our website helpful, please tell other aspiring nurses, nursing students, and professors. Please link to our site from your blogs, videos and college websites, or share us on your favorite social media sites. Thank you for your support!”

👩‍⚕️ Join the Nurse RN lesson in progress LIVE 

NCLEX Question 3: A nurse is caring for a patient with suspected acute coronary syndrome (ACS). The patient complains of chest pain that began 20 minutes ago. Which action should the nurse prioritize first?
A) Administering aspirin
B) Obtaining a 12-lead ECG
C) Administering nitroglycerin
D) Starting an IV line

QBankproAcademy.com

Explanation: In a patient with suspected ACS and chest pain, obtaining a 12-lead ECG is the top priority as it allows for the immediate assessment of cardiac rhythm and ST-segment changes, helping to confirm the diagnosis and guide treatment decisions. Administering aspirin (option A) and nitroglycerin (option C) are important interventions, but they should be based on the ECG findings and provider’s orders. Starting an IV line (option D) can also be important but is secondary to obtaining the ECG to confirm the diagnosis.

 
FREE nclex practice questions
Click “Join Us”: Become a Member – QBankProAcademy.com

NCLEX Question 4: A 55-year-old patient is admitted with chest pain and is diagnosed with unstable angina. The nurse anticipates which medication will likely be prescribed to manage this condition?
A) Warfarin (Coumadin)
B) Heparin
C) Lisinopril (Zestril)
D) Nitroglycerin

QBankproAcademy.com

Explanation: Nitroglycerin is commonly prescribed for the management of unstable angina because it helps relax coronary arteries, increase blood flow to the heart, and relieve chest pain. Warfarin (option A) and heparin (option B) are anticoagulants and are typically used for conditions like atrial fibrillation and deep vein thrombosis. Lisinopril (option C) is an ACE inhibitor used to manage blood pressure and heart failure.
 

Struggling with pharm? Take me to “CRUSH the Pharm” questions!

NCLEX Question 5: A 60-year-old patient presents to the emergency department with sudden-onset chest pain, diaphoresis, and shortness of breath. The nurse suspects a myocardial infarction (MI). Which diagnostic test should be prioritized for this patient?
A) Chest X-ray
B) Complete blood count (CBC)
C) 12-lead electrocardiogram (ECG)
D) Arterial blood gases (ABGs)

QBankproAcademy.com

Explanation: The 12-lead ECG is the most important diagnostic test to prioritize when a myocardial infarction (MI) is suspected. It helps identify ST-segment changes and other ECG abnormalities indicative of MI. While other tests may be ordered later for further assessment (e.g., CBC, ABGs, or chest X-ray), the 12-lead ECG is critical for rapid diagnosis and treatment.

Nursing Pharmacology Flashcards

NCLEX Question 6: A patient with a history of heart failure is prescribed a diuretic to manage fluid retention. What potential side effect should the nurse educate the patient about and monitor for?
A) Hypertension
B) Hypokalemia
C) Bradycardia
D) Hyperglycemia

QBankproAcademy.com

Explanation: Diuretics can lead to electrolyte imbalances, including hypokalemia (low potassium levels), which can be particularly concerning for patients with heart failure. The nurse should educate the patient about signs and symptoms of hypokalemia and monitor potassium levels regularly. Hypertension (option A), bradycardia (option C), and hyperglycemia (option D) are not typically associated with diuretic use but may occur with other medications or conditions.

NCLEX Question 7: A 45-year-old patient is scheduled for coronary artery bypass grafting (CABG) surgery. Which nursing intervention is most important in the immediate preoperative period?
A) Administering pain medication
B) Providing emotional support
C) Verifying informed consent
D) Assessing the patient’s dietary preferences

QBankproAcademy.com

Explanation: In the immediate preoperative period, the most critical nursing intervention is verifying that the patient has given informed consent for the surgical procedure. This ensures that the patient fully understands the risks, benefits, and alternatives of the surgery and has made an informed decision. While providing emotional support (option B) is important, verifying informed consent takes precedence. Administering pain medication (option A) and assessing dietary preferences (option D) are not the top priorities in this context.

NCLEX Flashcards: Blood Pressure MedsCardiovascular Meds | Renal System Meds | Gastrointestinal MedsRespiratory Meds

NCLEX Question 8: A 62-year-old patient is admitted to the hospital with a diagnosis of heart failure. The nurse understands that which of the following symptoms is characteristic of left-sided heart failure?
A) Peripheral edema
B) Jugular vein distention
C) Ascites
D) Crackles in the lungs

QBankproAcademy.com

Explanation: Crackles in the lungs are a characteristic symptom of left-sided heart failure due to the accumulation of fluid in the pulmonary tissues and alveoli. Peripheral edema (option A) is more indicative of right-sided heart failure. Jugular vein distention (option B) may be seen in right-sided heart failure as well. Ascites (option C) is also more commonly associated with right-sided heart failure.

NCLEX Question 9: A 50-year-old patient is prescribed atorvastatin (Lipitor) to manage high cholesterol levels. The nurse should educate the patient about the importance of regular monitoring of which laboratory value while taking this medication?
A) Serum potassium
B) Serum creatinine
C) Serum glucose
D) Serum liver enzymes

QBankproAcademy.com

Explanation: Atorvastatin and other statin medications can affect liver function, so it’s important to monitor serum liver enzymes regularly to detect any signs of liver damage. Monitoring serum potassium (option A) is not specific to statin therapy. Serum creatinine (option B) is important for kidney function assessment but not directly related to statin use. Serum glucose (option C) is not routinely monitored for statin therapy.

Focus on NCLEX Cardiovascular Questions

NCLEX Question 10: A 68-year-old patient with a history of hypertension and diabetes is being evaluated for their cardiovascular risk. The nurse assesses the patient’s blood pressure, which measures 150/90 mm Hg. How should the nurse classify this patient’s blood pressure reading?
A) Normal
B) Prehypertension
C) Hypertension Stage 1
D) Hypertension Stage 2

QBankproAcademy.com

Explanation: A blood pressure reading of 150/90 mm Hg falls within the prehypertension range, indicating an increased risk of developing hypertension. Classification as normal (option A) would require a reading of less than 120/80 mm Hg. Hypertension Stage 1 (option C) typically starts at 140/90 mm Hg, and Hypertension Stage 2 (option D) usually begins at 160/100 mm Hg or higher.

👩‍⚕️Want to get FREE email updates from QBANKPROACADEMY.COM about free printables and NEW Nursing Tips? Sign up below!

NCLEX Question 11: A 55-year-old male patient presents to the emergency department with severe chest pain radiating to his left arm. He is diaphoretic and anxious. An electrocardiogram (ECG) shows ST-segment elevation in multiple leads. The nurse recognizes these findings as indicative of:
A) Stable angina
B) Unstable angina
C) Non-ST-elevation myocardial infarction (NSTEMI)
D) ST-elevation myocardial infarction (STEMI)

Explanation: The ST-segment elevation in multiple leads on the ECG is a classic sign of a STEMI, which indicates an acute and complete blockage of a coronary artery. STEMI is a medical emergency requiring immediate intervention, such as percutaneous coronary intervention (PCI) or thrombolytic therapy.

NCLEX Question 12: Case: A 62-year-old female patient is admitted with a diagnosis of heart failure. She complains of shortness of breath, orthopnea, and bilateral lower extremity edema. The nurse should anticipate that the healthcare provider will prescribe which medication to manage her heart failure symptoms?
A) Furosemide (Lasix)
B) Warfarin (Coumadin)
C) Aspirin
D) Albuterol inhaler

QBankproAcademy.com

Explanation: Furosemide, a loop diuretic, is commonly prescribed to manage heart failure symptoms by reducing fluid overload and congestion. Warfarin (option B) may be used for certain heart conditions but is not primarily for symptom relief. Aspirin (option C) is more commonly used for antiplatelet therapy. An albuterol inhaler (option D) is used to manage respiratory conditions and is not a first-line treatment for heart failure.

EKG showing hyperkalemia. NCLEX, ANPP, ANCC Questions and Answers
Hyperkalemia: ECG changes

NCLEX Question 13: Case: A 45-year-old patient with a family history of cardiovascular disease is seen in the clinic for a routine check-up. The nurse emphasizes the importance of which lifestyle modification to reduce the patient’s cardiovascular risk?
A) Increasing saturated fat intake
B) Smoking cessation
C) Reducing fruit and vegetable consumption
D) Limiting physical activity

QBankproAcademy.com

Explanation: Smoking cessation is a crucial lifestyle modification to reduce cardiovascular risk. Smoking is a significant risk factor for heart disease. Options A, C, and D are detrimental to heart health and should be discouraged. Increasing saturated fat intake (option A) and reducing fruit and vegetable consumption (option C) are associated with an unhealthy diet, while limiting physical activity (option D) is detrimental to overall cardiovascular health.

NCLEX questions by topic: RSV, Pediatrics, Pharmacology, IV Fluids, Metronidazole, Cardiac, Cushings, Brain, Neurological Disorders, Cardiovascular, Childbirth, COPD, Depression, Endocrine, Adrenal Disorders, Kidney Disease

NCLEX Question 14: A 58-year-old patient with a history of coronary artery disease (CAD) is prescribed clopidogrel (Plavix) following a recent stent placement. The nurse should educate the patient about the purpose of clopidogrel, which is primarily to:
A) Lower blood pressure
B) Prevent platelet aggregation and blood clot formation
C) Improve cholesterol levels
D) Manage chest pain

QBankproAcademy.com

Explanation: Clopidogrel is an antiplatelet medication commonly prescribed to prevent platelet aggregation and reduce the risk of blood clot formation, especially in patients with CAD or following procedures like stent placement. It does not lower blood pressure (option A), improve cholesterol levels (option C), or manage chest pain (option D).

NCLEX Question 15: A 68-year-old patient with heart failure is prescribed digoxin. The nurse should monitor for which potential side effect of digoxin toxicity?
A) Hypertension
B) Hyperkalemia
C) Bradycardia
D) Respiratory distress

QBankproAcademy.com

Explanation: Digoxin toxicity can lead to bradycardia (slow heart rate) as one of its adverse effects. Hypertension (option A), hyperkalemia (option B), and respiratory distress (option D) are not commonly associated with digoxin toxicity.

NCLEX Question 16: A 55-year-old patient with a history of hypertension and diabetes is seen in the clinic for a follow-up appointment. The nurse educates the patient about the importance of controlling blood pressure and blood glucose levels to reduce the risk of cardiovascular complications. Which target blood pressure range should the nurse recommend for this patient?
A) Less than 120/80 mm Hg
B) Less than 140/90 mm Hg
C) Less than 160/100 mm Hg
D) Less than 180/110 mm Hg

QBankproAcademy.com

Explanation: The recommended target blood pressure for most patients with hypertension, including those with diabetes, is less than 140/90 mm Hg. Option A represents the normal blood pressure range. Options C and D indicate uncontrolled hypertension.

NCLEX Question 17: A 60-year-old patient with a history of dyslipidemia is prescribed simvastatin to manage high cholesterol levels. The nurse should instruct the patient to take simvastatin at which time of day for optimal effectiveness?
A) Morning
B) Noon
C) Evening
D) Bedtime

QBankproAcademy.com

Explanation: Statin medications like simvastatin are often recommended to be taken in the evening because cholesterol production in the liver tends to be highest during nighttime hours. Taking the medication at this time can optimize its effectiveness in reducing cholesterol levels. Morning (option A), noon (option B), and bedtime (option D) may not be as effective in achieving this goal.

Respiratory Problems

NCLEX Question 18: A 65-year-old patient with a history of chronic obstructive pulmonary disease (COPD) presents to the emergency department with increased dyspnea, cough, and greenish sputum production. The nurse suspects an acute exacerbation of COPD. Which intervention should the nurse prioritize?
A) Administering a high-flow oxygen mask
B) Administering a bronchodilator medication
C) Placing the patient in a semi-Fowler’s position
D) Administering an antibiotic as ordered

QBankproAcademy.com

Explanation: In patients with an acute exacerbation of COPD, one of the primary concerns is the risk of bacterial infection. Antibiotics are often prescribed to treat or prevent infection in these patients, as infection can worsen the respiratory symptoms. Oxygen therapy, bronchodilators, and positioning are also important interventions, but addressing the potential infection is the priority.

NCLEX Question 19: A 40-year-old patient with a history of asthma is admitted to the hospital with severe respiratory distress. The patient is using accessory muscles, has audible wheezing, and is hypoxic. Which medication should the nurse anticipate administering first?
A) A corticosteroid
B) A short-acting beta-agonist (SABA)
C) An anticholinergic inhaler
D) A leukotriene modifier

QBankproAcademy.com

Explanation: In a patient with severe respiratory distress due to asthma, the initial treatment should focus on relieving bronchoconstriction and improving airflow. Short-acting beta-agonists (SABAs) like albuterol are the first-line medications for this purpose, as they quickly relax the airway smooth muscles, providing rapid relief. Corticosteroids, anticholinergics, and leukotriene modifiers are typically used as adjunct therapies or for long-term control.

NCLEX Question 20: A nurse is caring for a postoperative patient who has just undergone a lobectomy for lung cancer. Which assessment finding should the nurse report immediately to the healthcare provider?
A) Temperature of 100.2°F (37.9°C)
B) Decreased breath sounds on the affected side
C) Pain at the incision site rated 5 on a scale of 0-10
D) Sputum production with pink-tinged streaks

QBankproAcademy.com

Explanation: After a lobectomy, it is essential to monitor for any signs of respiratory complications, such as pneumothorax or atelectasis. Decreased or absent breath sounds on the affected side can indicate these issues and should be reported immediately to the healthcare provider for further evaluation and intervention. While the other options may require attention, decreased breath sounds are the most concerning in this context.

NCLEX Question 21: A 55-year-old patient with a history of chronic obstructive pulmonary disease (COPD) is experiencing severe dyspnea. The nurse observes paradoxical chest movement during inspiration. Which of the following interventions should the nurse prioritize?
A) Administering a bronchodilator medication
B) Initiating mechanical ventilation
C) Placing the patient in a high-Fowler’s position
D) Administering supplemental oxygen

QBankproAcademy.com

Explanation: Paradoxical chest movement, where the chest wall moves inward during inspiration instead of outward, is a sign of severe respiratory distress. In this case, the patient may be experiencing respiratory muscle fatigue or failure. Initiating mechanical ventilation is crucial to provide adequate support for breathing and ensure oxygenation. While bronchodilators, positioning, and oxygen may also be necessary, they do not address the primary concern of respiratory failure.

NCLEX Question 22: A 30-year-old patient with a known peanut allergy accidentally ingests peanuts and develops severe respiratory distress. The nurse recognizes the signs of anaphylaxis. What is the nurse’s immediate priority action?
A) Administering an antihistamine
B) Administering epinephrine (adrenaline)
C) Administering corticosteroids
D) Initiating oxygen therapy

QBankproAcademy.com

Explanation: In anaphylactic reactions, the immediate priority is to administer epinephrine (adrenaline) to counteract the severe allergic response, which includes bronchoconstriction and airway swelling. Epinephrine helps reverse these effects and should be administered as soon as possible. Antihistamines and corticosteroids may be used as adjunct therapies, but they are not the first-line treatment for anaphylaxis. Oxygen therapy can be initiated after administering epinephrine if needed.

NCLEX Question 23: A 45-year-old patient with a history of asthma presents to the emergency department with acute wheezing, tachypnea, and accessory muscle use. The nurse initiates the administration of a bronchodilator. Which bronchodilator medication is commonly used as the first-line treatment for acute asthma exacerbations?
A) Montelukast (Singulair)
B) Ipratropium bromide (Atrovent)
C) Beclomethasone (QVAR)
D) Albuterol (Ventolin)

QBankproAcademy.com

Explanation: Albuterol (Ventolin) is a short-acting beta-agonist (SABA) and is the first-line bronchodilator medication for acute asthma exacerbations. It rapidly relaxes bronchial smooth muscles and provides quick relief from bronchoconstriction and wheezing. Montelukast is a leukotriene modifier, ipratropium bromide is an anticholinergic inhaler, and beclomethasone is an inhaled corticosteroid; these medications are typically used for long-term asthma management or as adjunct therapy, not for the rapid relief of acute symptoms.

NCLEX Question 24: A 68-year-old patient with a history of chronic bronchitis is receiving oxygen therapy via nasal cannula at 2 liters per minute (LPM). The nurse notes that the patient’s oxygen saturation (SpO2) has dropped to 88%. What should the nurse do first?
A) Increase the oxygen flow rate to 4 LPM
B) Administer a bronchodilator medication
C) Encourage the patient to take deep breaths
D) Notify the healthcare provider

QBankproAcademy.com

Explanation: A drop in oxygen saturation (SpO2) indicates inadequate oxygen delivery. The first action should be to increase the oxygen flow rate to improve oxygenation. This can help raise the SpO2 to a more acceptable level. Administering bronchodilators, encouraging deep breaths, and notifying the healthcare provider may also be necessary but are secondary interventions in this situation.

NCLEX Question 25: A 55-year-old patient has been diagnosed with pneumonia and is receiving antibiotic therapy. During the assessment, the nurse notes increased tactile fremitus and dullness to percussion over the right lower lung field. Which condition should the nurse suspect?
A) Pleural effusion
B) Atelectasis
C) Pneumothorax
D) Pulmonary embolism

QBankproAcademy.com

Explanation: Increased tactile fremitus and dullness to percussion over a lung field are indicative of a pleural effusion. A pleural effusion is an accumulation of fluid in the pleural space, which can compress the lung tissue and lead to these findings. Atelectasis, pneumothorax, and pulmonary embolism may present with different clinical signs and symptoms.

NCLEX Question 26: A 30-year-old patient with a history of smoking is admitted with acute shortness of breath, chest pain, and hemoptysis (coughing up blood). The healthcare provider suspects a pulmonary embolism (PE). What diagnostic test should the nurse anticipate being ordered to confirm the diagnosis of PE?
A) Chest X-ray
B) Electrocardiogram (ECG)
C) D-dimer blood test
D) Arterial blood gases (ABG) analysis

QBankproAcademy.com

Explanation: A D-dimer blood test is a screening tool used to assess for the presence of abnormal blood clotting, such as in cases of pulmonary embolism (PE). Elevated D-dimer levels can indicate the presence of a clot, but it is not confirmatory on its own. Other diagnostic tests, such as a CT pulmonary angiography or ventilation-perfusion (V/Q) scan, are typically ordered to confirm the diagnosis of PE. Chest X-ray, ECG, and ABG analysis may provide information but are not the primary tests for diagnosing PE.

NCLEX Question 27: A 50-year-old patient with a history of asthma is prescribed a peak flow meter to monitor respiratory function at home. Which action should the nurse recommend for proper use of the peak flow meter?
A) Inhale as deeply as possible before blowing into the meter
B) Stand while using the peak flow meter for accurate results
C) Record the highest of three consecutive readings each time
D) Use the peak flow meter only when experiencing severe asthma symptoms

QBankproAcademy.com

Explanation: To ensure accurate peak flow meter readings, patients should be instructed to record the highest of three consecutive readings. This helps account for variability in performance and provides a more reliable assessment of lung function. Inhaling deeply before blowing into the meter is not necessary and can lead to incorrect readings. Patients can sit or stand while using the peak flow meter, but consistency in position is key. Regular monitoring, not just during severe symptoms, is important for asthma management.

NCLEX Question 28: A 60-year-old patient is admitted with a suspected pulmonary embolism (PE). The nurse is preparing to administer anticoagulant therapy. What medication is commonly used to treat acute PE by preventing further clot formation?
A) Warfarin (Coumadin)
B) Clopidogrel (Plavix)
C) Heparin
D) Aspirin

QBankproAcademy.com

Explanation: Heparin is commonly used to treat acute pulmonary embolism (PE) by preventing further clot formation. It has a rapid onset of action and can be administered intravenously to quickly reduce the risk of clot progression. Warfarin is used for long-term anticoagulation, while clopidogrel and aspirin are typically used to prevent platelet aggregation and are not the primary treatment for acute PE.

NCLEX Question 29: A 45-year-old patient is receiving mechanical ventilation due to respiratory failure. The nurse observes the patient’s heart rate increasing from 80 to 120 beats per minute, and the patient appears anxious. What intervention should the nurse prioritize?
A) Administering a sedative medication
B) Decreasing the ventilator rate and tidal volume
C) Assessing for pain or discomfort
D) Increasing the fraction of inspired oxygen (FiO2)

QBankproAcademy.com

Explanation: In a mechanically ventilated patient, an increase in heart rate and signs of anxiety may indicate pain or discomfort. It is crucial to assess the patient’s pain level and discomfort and address any underlying causes. Administering a sedative should not be the first intervention, as it may mask the underlying issue. Adjusting ventilator settings (decreasing rate and tidal volume) or increasing FiO2 may be necessary but should come after assessing and addressing the patient’s comfort and pain.

NCLEX Question 30: A 65-year-old patient with a history of emphysema presents to the clinic with complaints of increased dyspnea, chronic cough, and thick sputum production. Which medication should the nurse expect the healthcare provider to prescribe to improve the patient’s sputum clearance?
A) Albuterol (Ventolin)
B) Mon: telukast (Singulair)
C) Ipratropium bromide (Atrovent)
D) Guaifenesin (Mucinex)

QBankproAcademy.com

Explanation: Guaifenesin (Mucinex) is an expectorant that can help improve sputum clearance by thinning and loosening mucus secretions, making them easier to cough up. Albuterol, Montelukast, and Ipratropium bromide are bronchodilator medications and are typically used to relieve bronchoconstriction and improve airflow but do not directly address sputum clearance.

NCLEX Question 31: A 50-year-old patient with a history of smoking is at risk for developing chronic obstructive pulmonary disease (COPD). Which nursing intervention is most effective in preventing COPD in this patient?
A) Administering bronchodilator medications regularly
B) Teaching smoking cessation strategies
C) Providing supplemental oxygen therapy
D) Recommending annual chest X-rays

QBankproAcademy.com

Explanation: Smoking is the most significant risk factor for COPD. Teaching smoking cessation strategies is the most effective intervention for preventing the development of COPD in a patient with a history of smoking. Bronchodilators, supplemental oxygen therapy, and annual chest X-rays may be components of COPD management or monitoring but do not prevent the condition’s development.

NCLEX Question 32: A 35-year-old patient presents with sudden-onset, severe chest pain that worsens with inspiration. The nurse suspects a pulmonary embolism (PE). Which diagnostic test is commonly used to confirm the presence of a PE?
A) Complete blood count (CBC)
B) Chest X-ray
C) D-dimer blood test
D) Arterial blood gases (ABG) analysis

QBankproAcademy.com

Explanation: The D-dimer blood test is a commonly used initial screening test to assess for the presence of abnormal blood clotting, such as in cases of pulmonary embolism (PE). Elevated D-dimer levels may suggest the presence of a clot, but it is not confirmatory on its own. Other diagnostic tests, such as a CT pulmonary angiography or ventilation-perfusion (V/Q) scan, are typically ordered to confirm or rule out PE. CBC, chest X-ray, and ABG analysis may provide additional information but are not the primary tests for diagnosing PE.

upper gastrointestinal Disorders

NCLEX Question 33: A 55-year-old patient presents to the emergency department with severe epigastric pain, which is relieved by eating or taking antacids. The patient reports a history of smoking and alcohol consumption. On physical examination, tenderness is noted in the epigastric area. Which of the following is the most likely diagnosis?
A) Diverticulitis
B) Gastroesophageal Reflux Disease (GERD)
C) Peptic Ulcer Disease (PUD) 
D) Crohn’s Disease

QBankproAcademy.com

Explanation: This patient’s symptoms, including epigastric pain relieved by eating and a history of smoking and alcohol consumption, are indicative of Peptic Ulcer Disease (PUD). Tenderness in the epigastric area is a common finding in PUD. Diverticulitis, GERD, and Crohn’s Disease typically present with different symptoms and findings.

NCLEX Question 34: A 45-year-old female patient complains of difficulty swallowing, a sensation of food getting stuck in her throat, and occasional regurgitation of undigested food. Which of the following conditions is the most likely cause of her symptoms?
A) Gastroenteritis
B) Hiatal Hernia
C) Inflammatory Bowel Disease (IBD)
D) Gastric Ulcer

QBankproAcademy.com

Explanation: The patient’s symptoms of difficulty swallowing, sensation of food getting stuck, and regurgitation of undigested food are characteristic of Hiatal Hernia, where a portion of the stomach protrudes through the diaphragm into the chest cavity. Gastroenteritis, IBD, and Gastric Ulcer typically present with different symptoms.

Read more about Crohn’s disease and ulcerative colitis

NCLEX Question 35: A 65-year-old male presents with unintentional weight loss, dysphagia, and a chronic cough. Barium swallow shows a “bird beak” appearance of the esophagus, and manometry reveals impaired lower esophageal sphincter relaxation. Which of the following conditions is the likely diagnosis?
A) Gastroesophageal Reflux Disease (GERD)
B) Achalasia
C) Peptic Ulcer Disease (PUD)
D) Esophageal Cancer

QBankproAcademy.com

Explanation: The “bird beak” appearance on barium swallow and impaired lower esophageal sphincter relaxation on manometry are characteristic findings of Achalasia, a motility disorder of the esophagus. This condition can lead to dysphagia, chronic cough, and unintentional weight loss. GERD, PUD, and Esophageal Cancer typically present with different clinical and diagnostic features.

NCLEX Question 36: A 45-year-old male patient presents with recurrent burning chest pain that worsens after meals and when lying down. The pain is relieved by antacids. Which of the following is the most likely diagnosis?
A) Irritable Bowel Syndrome (IBS)
B) Gastroesophageal Reflux Disease (GERD)
C) Inflammatory Bowel Disease (IBD)
D) Gastric Ulcer

QBankproAcademy.com

Explanation: The patient’s symptoms of burning chest pain worsened after meals and improved with antacids are typical of GERD, a condition characterized by the backward flow of stomach acid into the esophagus. IBS, IBD, and Gastric Ulcer typically present with different symptoms.

NCLEX Question 37: A 60-year-old female patient presents with chronic abdominal pain, especially in the upper left quadrant. She experiences early satiety, bloating, and nausea. Physical examination reveals an enlarged spleen. Which of the following conditions is the most likely cause of her symptoms?
A) Diverticulitis
B) Peptic Ulcer Disease (PUD)
C) Gastric Cancer
D) Splenomegaly

QBankproAcademy.com

Explanation: The patient’s symptoms, including chronic abdominal pain, early satiety, bloating, nausea, and an enlarged spleen, raise suspicion for Gastric Cancer. These symptoms can be associated with advanced stages of gastric cancer. Diverticulitis, PUD, and splenomegaly typically present with different clinical features.

NCLEX Question 38: A 50-year-old male patient presents with severe, intermittent, crampy abdominal pain, diarrhea, and weight loss. He reports a history of smoking. Colonoscopy reveals skip lesions with transmural inflammation. Which of the following conditions is the most likely diagnosis?
A) Irritable Bowel Syndrome (IBS)
B) Crohn’s Disease
C) Gastroesophageal Reflux Disease (GERD)
D) Gastroenteritis

QBankproAcademy.com

Explanation: The patient’s symptoms of crampy abdominal pain, diarrhea, weight loss, smoking history, and skip lesions with transmural inflammation seen on colonoscopy are consistent with Crohn’s Disease, an inflammatory bowel disease. IBS, GERD, and Gastroenteritis typically present with different clinical and diagnostic features.

NCLEX Question 39: A 45-year-old patient presents with a gnawing, burning epigastric pain that occurs between meals and often wakes them up at night. The pain is relieved by antacids and food. Which of the following conditions is the most likely diagnosis?
A) Diverticulitis
B) Gastroesophageal Reflux Disease (GERD)
C) Peptic Ulcer Disease (PUD)
D) Inflammatory Bowel Disease (IBD)

QBankproAcademy.com

Explanation: This patient’s symptoms, including epigastric pain relieved by antacids and food, are characteristic of Peptic Ulcer Disease (PUD). PUD is often associated with the presence of Helicobacter pylori infection or NSAID use. Diverticulitis, GERD, and IBD typically present with different symptoms.

NCLEX Question 40: A 55-year-old male patient complains of difficulty swallowing solid foods, frequent heartburn, and regurgitation of food. On endoscopy, there are erosions and inflammation in the lower esophagus. Which of the following conditions is the most likely diagnosis?
A) Hiatal Hernia
B) Gastric Ulcer
C) Esophageal Cancer
D) Gastroesophageal Reflux Disease (GERD)

QBankproAcademy.com

Explanation: The patient’s symptoms, including difficulty swallowing, heartburn, regurgitation, and endoscopic findings of erosions and inflammation in the lower esophagus, are indicative of GERD. GERD occurs due to the reflux of stomach acid into the esophagus, leading to these symptoms. Hiatal Hernia, Gastric Ulcer, and Esophageal Cancer typically present with different clinical features.

NCLEX Question 41: A 65-year-old female patient presents with progressive dysphagia, unintentional weight loss, and a feeling of food getting stuck in the throat. On barium swallow, there is a narrowing of the esophagus with a “bird beak” appearance. What is the most likely diagnosis?
A) Gastroenteritis
B) Achalasia
C) Peptic Ulcer Disease (PUD)
D) Crohn’s Disease

QBankproAcademy.com

Explanation: The patient’s symptoms of dysphagia, weight loss, and “bird beak” appearance on barium swallow are classic signs of achalasia, a motility disorder of the esophagus characterized by impaired esophageal peristalsis and failure of the lower esophageal sphincter to relax. Gastroenteritis, PUD, and Crohn’s Disease typically present with different clinical features.

NCLEX Question 42: A 50-year-old patient complains of burning chest pain that worsens when lying down or bending over. The pain is often accompanied by regurgitation of sour-tasting fluid into the mouth. Which of the following is the most likely diagnosis?
A) Irritable Bowel Syndrome (IBS)
B) Gastroesophageal Reflux Disease (GERD)
C) Inflammatory Bowel Disease (IBD)
D) Gastric Ulcer

QBankproAcademy.com

Explanation: This patient’s symptoms, including burning chest pain exacerbated by lying down and regurgitation, are characteristic of GERD, a condition where stomach acid flows back into the esophagus. IBS, IBD, and Gastric Ulcer typically present with different symptoms.

NCLEX Question 43: A 60-year-old male patient presents with upper abdominal pain that improves with food intake and worsens when the stomach is empty. He also reports dark, tarry stools. What condition is most likely responsible for these symptoms?
A) Hiatal Hernia
B) Gastroesophageal Reflux Disease (GERD)
C) Peptic Ulcer Disease (PUD)
D) Inflammatory Bowel Disease (IBD)

QBankproAcademy.com

Explanation: This patient’s symptoms of upper abdominal pain relieved by food intake and dark, tarry stools suggest Peptic Ulcer Disease (PUD). PUD can lead to bleeding ulcers, resulting in the passage of dark, tarry stools (melena). Hiatal Hernia, GERD, and IBD typically present with different clinical features.

NCLEX Question 44: A 55-year-old female patient presents with difficulty swallowing, unintentional weight loss, and a sensation of food getting stuck in her throat. On endoscopy, there are narrowing and strictures in the esophagus. What condition is most likely responsible for these findings?
A) Gastroenteritis
B) Achalasia
C) Gastroesophageal Reflux Disease (GERD)
D) Crohn’s Disease

QBankproAcademy.com

Explanation: The patient’s symptoms of difficulty swallowing, weight loss, and endoscopic findings of narrowing and strictures in the esophagus are suggestive of Achalasia, a motility disorder of the esophagus characterized by impaired esophageal peristalsis and failure of the lower esophageal sphincter to relax. Gastroenteritis, GERD, and Crohn’s Disease typically present with different clinical features.

NCLEX Question 45: A 40-year-old patient presents with severe upper abdominal pain that radiates to the back, accompanied by nausea and vomiting. On physical examination, there is tenderness in the upper abdomen. Laboratory tests reveal elevated serum amylase and lipase levels. Which of the following conditions is the most likely diagnosis?
A) Gastroesophageal Reflux Disease (GERD)
B) Peptic Ulcer Disease (PUD)
C) Cholecystitis
D) Pancreatitis

QBankproAcademy.com

Explanation: The patient’s symptoms of severe upper abdominal pain radiating to the back, along with elevated serum amylase and lipase levels, are indicative of acute pancreatitis, which can result from various causes, including gallstones and alcohol consumption. GERD, PUD, and cholecystitis typically present with different symptoms and laboratory findings.

NCLEX Question 46: A 55-year-old male patient with a history of smoking presents with progressive dysphagia, unintentional weight loss, and hoarseness. On endoscopy, there is a visible mass in the esophagus. What is the most likely diagnosis?
A) Hiatal Hernia
B) Gastroesophageal Reflux Disease (GERD)
C) Esophageal Cancer
D) Gastroenteritis

QBankproAcademy.com

Explanation: The patient’s symptoms of dysphagia, weight loss, hoarseness, and the presence of a visible mass on endoscopy are concerning for esophageal cancer, which is often associated with risk factors such as smoking and alcohol consumption. Hiatal Hernia, GERD, and Gastroenteritis typically present with different clinical features.

NCLEX Question 47: A 50-year-old female patient presents with recurrent upper abdominal pain that worsens after meals and is relieved by antacids. She also reports a history of frequent use of nonsteroidal anti-inflammatory drugs (NSAIDs). Which of the following conditions is the most likely diagnosis?
A) Diverticulitis
B) Gastroesophageal Reflux Disease (GERD)
C) Peptic Ulcer Disease (PUD)
D) Inflammatory Bowel Disease (IBD)

QBankproAcademy.com

Explanation: The patient’s symptoms, including upper abdominal pain worsened after meals and relieved by antacids, along with a history of frequent NSAID use, are suggestive of Peptic Ulcer Disease (PUD). PUD can be caused by factors like NSAID use and Helicobacter pylori infection. Diverticulitis, GERD, and IBD typically present with different symptoms and risk factors.

lower gastrointestinal Disorders

NCLEX Question 48: Which clinical manifestation is most indicative of diverticulitis in a patient with lower gastrointestinal disease?
A) Hematochezia (bright red blood in stool)
B) Diarrhea with mucus and abdominal cramping
C) Left lower quadrant pain with fever and leukocytosis
D) Steatorrhea (foul-smelling, bulky, greasy stools)

QBankproAcademy.com

Explanation: Diverticulitis is characterized by inflammation or infection of diverticula in the colon, commonly presenting with left lower quadrant abdominal pain, fever, and leukocytosis due to the inflammatory response. Hematochezia (A) is more indicative of lower gastrointestinal bleeding, while diarrhea with mucus and abdominal cramping (B) might be seen in conditions like irritable bowel syndrome. Steatorrhea (D) is associated with malabsorption disorders, not diverticulitis.

NCLEX Question 49: A patient with Crohn’s disease presents with recurrent abdominal pain, diarrhea, and weight loss. Upon examination, which finding is consistent with this inflammatory bowel disease?
A) Cobblestone appearance on colonoscopy
B) Isolated involvement of the rectum
C) Elevation of serum amylase and lipase levels
D) Microscopic hematuria on urinalysis

QBankproAcademy.com

Explanation: Crohn’s disease often presents with a cobblestone appearance on colonoscopy (A), characterized by patchy areas of inflamed and ulcerated mucosa separated by areas of normal tissue. It can affect any part of the gastrointestinal tract, not just the rectum. Elevated serum amylase and lipase levels (C) are more indicative of pancreatitis, and microscopic hematuria (D) suggests a kidney or urinary tract problem, not Crohn’s disease.

NCLEX Question 50: A nurse is caring for a patient with colorectal cancer who underwent surgical resection. Which postoperative complication should the nurse prioritize monitoring for in this patient?
A) Urinary tract infection
B) Incisional hernia
C) Deep vein thrombosis
D) Increased intracranial pressure

QBankproAcademy.com

Explanation: Patients undergoing colorectal surgery are at an increased risk of developing deep vein thrombosis (DVT) due to immobility and venous stasis. Therefore, monitoring for signs of DVT, such as leg swelling, pain, or redness, is crucial for early detection and intervention. While urinary tract infections (A) and incisional hernias (B) are potential complications, they are not as commonly associated with colorectal surgery. Increased intracranial pressure (D) is unrelated to colorectal cancer surgery.

NCLEX Question 51: A patient with a history of gastroesophageal reflux disease (GERD) presents with heartburn, regurgitation, and a sensation of food sticking in the throat. Which diagnostic test is most appropriate to confirm the presence of GERD?
A) Complete blood count (CBC)
B) Esophagogastroduodenoscopy (EGD)
C) Chest X-ray
D) Urine pH monitoring

QBankproAcademy.com

Explanation: Esophagogastroduodenoscopy (EGD) (B) is the gold standard diagnostic test for GERD as it allows direct visualization of the esophagus and assessment of mucosal damage. A CBC (A) is not specific to GERD and would not confirm the diagnosis. Chest X-ray (C) and urine pH monitoring (D) are not typically used for diagnosing GERD.

NCLEX Question 52: A patient presents with severe epigastric pain that radiates to the back, nausea, and vomiting. Physical examination reveals tenderness to palpation in the upper abdomen. Which condition is most likely responsible for these symptoms?
A) Gastroenteritis
B) Peptic ulcer disease (PUD)
C) Cholecystitis
D) Appendicitis

QBankproAcademy.com

Explanation: The symptoms described, including severe epigastric pain radiating to the back, nausea, vomiting, and upper abdominal tenderness, are characteristic of peptic ulcer disease (B). Gastroenteritis (A) typically presents with diarrhea and may involve lower abdominal pain. Cholecystitis (C) presents with right upper quadrant pain, and appendicitis (D) typically presents with lower right abdominal pain.

NCLEX Question 53: A nurse is caring for a patient with a known history of esophageal varices. Which intervention should the nurse prioritize to prevent bleeding in this patient?
A) Administering proton pump inhibitors (PPIs)
B) Administering antispasmodic medications
C) Administering stool softeners
D) Administering beta-blockers

QBankproAcademy.com

Explanation: To prevent bleeding from esophageal varices, the nurse should prioritize administering beta-blockers (D), such as propranolol or nadolol, to reduce portal pressure and lower the risk of variceal rupture. Proton pump inhibitors (PPIs) (A) may help in managing gastroesophageal reflux but are not the primary intervention for esophageal varices. Antispasmodic medications (B) are not typically used for varices. Stool softeners (C) are unrelated to preventing variceal bleeding.

NCLEX Question 54: A patient is admitted with severe upper abdominal pain that radiates to the right shoulder, along with nausea and vomiting. Laboratory tests show elevated serum amylase and lipase levels. Which condition is the most likely cause of these symptoms?
A) Gastroesophageal reflux disease (GERD)
B) Cholecystitis
C) Peptic ulcer disease (PUD)
D) Irritable bowel syndrome (IBS)

QBankproAcademy.com

Explanation: The symptoms described, including severe upper abdominal pain radiating to the right shoulder, nausea, vomiting, and elevated serum amylase and lipase levels, are indicative of cholecystitis (B), which is inflammation of the gallbladder. This condition is commonly associated with the presence of gallstones. GERD (A), PUD (C), and IBS (D) do not typically present with these specific symptoms or laboratory findings.

NCLEX Question 55: A patient with a history of chronic alcohol abuse presents with epigastric pain, nausea, and vomiting that worsen after meals. The patient also experiences unintentional weight loss. Which condition is most likely responsible for these symptoms?
A) Pancreatitis
B) Gastric cancer
C) Peptic ulcer disease (PUD)
D) Gastroesophageal reflux disease (GERD)

QBankproAcademy.com

Explanation: The combination of epigastric pain, nausea, vomiting, unintentional weight loss, and a history of chronic alcohol abuse raises suspicion for gastric cancer (B). This malignancy often presents with vague upper gastrointestinal symptoms. While pancreatitis (A), PUD (C), and GERD (D) can cause some of these symptoms, they do not typically involve significant weight loss, especially in the context of alcohol abuse.

NCLEX Question 56: A nurse is caring for a patient with suspected gastroesophageal reflux disease (GERD). Which lifestyle modification should the nurse recommend to help alleviate the patient’s symptoms?
A) Increase dietary intake of spicy foods
B) Lie down immediately after meals
C) Elevate the head of the bed during sleep
D) Consume large meals close to bedtime

QBankproAcademy.com

Explanation: Elevating the head of the bed during sleep (C) is a lifestyle modification that can help reduce nighttime reflux symptoms in patients with GERD. It promotes gravity-assisted drainage of gastric contents away from the esophagus. Recommending the avoidance of spicy foods (A), lying down immediately after meals (B), or consuming large meals close to bedtime (D) would exacerbate GERD symptoms and should be discouraged.

NCLEX Question 57: A patient presents with a burning sensation in the upper abdomen, especially when hungry and at night. The discomfort improves with antacids. Which condition is most likely responsible for these symptoms?
A) Gastric cancer
B) Gastroesophageal reflux disease (GERD)
C) Peptic ulcer disease (PUD)
D) Irritable bowel syndrome (IBS)

QBankproAcademy.com

Explanation: The symptoms described, including a burning sensation in the upper abdomen that improves with antacids, are characteristic of peptic ulcer disease (PUD) (C). PUD is commonly associated with gastric and duodenal ulcers. Gastric cancer (A) typically presents with more severe and progressive symptoms. GERD (B) can cause heartburn but does not necessarily improve with antacids. IBS (D) primarily affects the lower gastrointestinal tract and is not characterized by upper abdominal burning.

NCLEX Question 58: A patient with a history of chronic NSAID use is admitted with melena (dark, tarry stools) and anemia. Which condition is most likely causing these symptoms?
A) Esophageal varices
B) Diverticulosis
C) Gastrointestinal bleeding due to NSAIDs
D) Crohn’s disease

QBankproAcademy.com

Explanation: Melena and anemia in a patient with a history of chronic NSAID use are highly suggestive of gastrointestinal bleeding, primarily from the stomach or duodenum (C). NSAIDs can irritate the gastric mucosa, leading to erosions or ulcers. Esophageal varices (A) may cause hematemesis (vomiting blood) but are less likely in this scenario. Diverticulosis (B) is associated with lower gastrointestinal bleeding. Crohn’s disease (D) may cause gastrointestinal bleeding but is not directly related to NSAID use.

NCLEX Question 59: A nurse is providing education to a patient recently diagnosed with gastroesophageal reflux disease (GERD). Which dietary recommendation should the nurse emphasize to help manage the patient’s symptoms?
A) Increase consumption of citrus fruits
B) Consume large meals close to bedtime
C) Avoid spicy and acidic foods
D) Eat high-fat, fried foods

QBankproAcademy.com

Explanation: Patients with GERD should avoid spicy and acidic foods (C) as these can exacerbate symptoms by increasing stomach acid production and irritating the esophagus. Citrus fruits (A) can also be acidic and should be consumed in moderation. Consuming large meals close to bedtime (B) can lead to reflux symptoms, so this should be discouraged. High-fat, fried foods (D) can relax the lower esophageal sphincter and worsen GERD symptoms, so they should also be avoided.

NCLEX Question 60: A patient presents with recurrent episodes of epigastric pain that occur 2-3 hours after meals and are relieved by eating or taking antacids. Which condition is most likely causing these symptoms?
A) Gastroesophageal reflux disease (GERD)
B) Gastric cancer
C) Peptic ulcer disease (PUD)
D) Irritable bowel syndrome (IBS)

QBankproAcademy.com

Explanation: The described symptoms, including epigastric pain that is relieved by eating or taking antacids, are characteristic of peptic ulcer disease (PUD) (C). PUD is often associated with gastric or duodenal ulcers, and pain typically occurs when the stomach is empty and gastric acid irritates the ulcer. GERD (A) can cause heartburn but does not necessarily improve with eating. Gastric cancer (B) typically presents with more severe and progressive symptoms. IBS (D) primarily affects the lower gastrointestinal tract and does not typically cause postprandial epigastric pain.

NCLEX Question 61: A patient complains of a chronic cough, hoarseness, and difficulty swallowing solid foods. Which upper gastrointestinal condition should the nurse suspect based on these symptoms?
A) Gastroesophageal reflux disease (GERD)
B) Peptic ulcer disease (PUD)
C) Gastric cancer
D) Cholecystitis

QBankproAcademy.com

Explanation: The symptoms described, including chronic cough, hoarseness, and difficulty swallowing solid foods, are indicative of complications associated with gastroesophageal reflux disease (GERD) (A). GERD can lead to irritation of the esophagus and result in these symptoms. PUD (B) primarily presents with abdominal pain. Gastric cancer (C) typically causes more advanced symptoms and may not initially manifest with these specific complaints. Cholecystitis (D) is unrelated to the symptoms described.

NCLEX Question 62: A patient presents with severe, sudden-onset, stabbing epigastric pain that radiates to the back, along with nausea and vomiting. The patient appears pale and diaphoretic. Which condition is most likely responsible for these symptoms?
A) Gastroesophageal reflux disease (GERD)
B) Peptic ulcer disease (PUD)
C) Acute pancreatitis
D) Crohn’s disease

QBankproAcademy.com

Explanation: The symptoms described, including severe, sudden-onset epigastric pain radiating to the back, nausea, vomiting, pallor, and diaphoresis, are characteristic of acute pancreatitis (C). Pancreatitis can cause intense abdominal pain and is often associated with gallstones or excessive alcohol consumption. GERD (A) does not typically present with these symptoms. PUD (B) may cause abdominal pain but is not associated with the same severity or radiation to the back. Crohn’s disease (D) primarily affects the lower gastrointestinal tract and does not typically cause sudden-onset epigastric pain.

NCLEX Question 63: A patient with lower gastrointestinal symptoms, including diarrhea, abdominal cramping, and bloody stools, is suspected to have inflammatory bowel disease (IBD). Which diagnostic test is most commonly used to differentiate between Crohn’s disease and ulcerative colitis?
A) Colonoscopy with biopsy
B) Abdominal ultrasound
C) Stool culture
D) Barium enema

QBankproAcademy.com

Explanation: To differentiate between Crohn’s disease and ulcerative colitis, the most common and definitive diagnostic test is a colonoscopy with biopsy (A). This procedure allows direct visualization of the gastrointestinal tract and the collection of tissue samples for histological examination. While other tests, such as abdominal ultrasound (B), stool culture (C), and barium enema (D), may provide valuable information, they are not as specific in distinguishing between the two types of IBD.

NCLEX Question 64: A patient presents with chronic constipation, abdominal distention, and infrequent passage of small, hard stools. The nurse suspects a lower gastrointestinal disorder. Which condition is most likely causing these symptoms?
A) Irritable bowel syndrome (IBS)
B) Diverticulitis
C) Crohn’s disease
D) Ulcerative colitis

QBankproAcademy.com

Explanation: The described symptoms, including chronic constipation, abdominal distention, and infrequent passage of small, hard stools, are characteristic of irritable bowel syndrome (IBS) (A). IBS can manifest as either diarrhea-predominant or constipation-predominant, and the patient’s symptoms align with the latter subtype. Diverticulitis (B), Crohn’s disease (C), and ulcerative colitis (D) are more commonly associated with diarrhea and are less likely to cause chronic constipation.

NCLEX Question 65: A nurse is caring for a patient with lower gastrointestinal bleeding. Which type of stool appearance is most indicative of lower gastrointestinal bleeding?
A) Bright red blood in stool
B) Dark, tarry stools (melena)
C) Fecal incontinence
D) Pencil-thin stools

QBankproAcademy.com

Explanation: Dark, tarry stools (melena) (B) are most indicative of lower gastrointestinal bleeding, as the blood has undergone digestion in the intestines, resulting in a black, tar-like appearance. Bright red blood in stool (A) suggests fresh bleeding from the lower gastrointestinal tract. Fecal incontinence (C) is a different symptom unrelated to the appearance of blood in the stool. Pencil-thin stools (D) can occur in various conditions but are not specific to gastrointestinal bleeding.

NCLEX Question 66: A patient presents with lower abdominal pain, diarrhea, and weight loss. Colonoscopy reveals skip lesions and cobblestone appearance of the mucosa with granulomas upon biopsy. Which condition is most likely responsible for these findings?
A) Diverticulitis
B) Irritable bowel syndrome (IBS)
C) Crohn’s disease
D) Ulcerative colitis

QBankproAcademy.com

Explanation: The described clinical presentation, including lower abdominal pain, diarrhea, weight loss, skip lesions, cobblestone appearance, and granulomas, is characteristic of Crohn’s disease (C). Crohn’s disease is an inflammatory bowel disease that can affect any part of the gastrointestinal tract, often resulting in patchy inflammation with skip lesions and characteristic mucosal changes. Diverticulitis (A) typically involves outpouchings in the colon, while IBS (B) is characterized by chronic abdominal pain with changes in bowel habits but does not typically cause granulomas. Ulcerative colitis (D) primarily affects the colon and presents with continuous inflammation and ulceration.

NCLEX Question 67: A patient with a history of chronic constipation presents with lower abdominal discomfort, bloating, and infrequent passage of small, pellet-like stools. Which lower gastrointestinal disorder should the nurse suspect in this case?
A) Diverticulosis
B) Irritable bowel syndrome (IBS)
C) Crohn’s disease
D) Ulcerative colitis

QBankproAcademy.com

Explanation: The patient’s symptoms, including chronic constipation, lower abdominal discomfort, bloating, and small pellet-like stools, are suggestive of irritable bowel syndrome (IBS) (B). IBS can manifest as either diarrhea-predominant or constipation-predominant, and this presentation aligns with the latter subtype. Diverticulosis (A) typically presents with intermittent lower abdominal pain and may involve diverticula in the colon. Crohn’s disease (C) and ulcerative colitis (D) are more commonly associated with diarrhea and are less likely to cause chronic constipation.

NCLEX Question 68: A nurse is caring for a patient with lower gastrointestinal bleeding. The patient’s stool is maroon in color and exhibits a foul odor. What should the nurse suspect as the likely source of bleeding?
A) Hemorrhoids
B) Diverticulosis
C) Gastric ulcers
D) Esophageal varices

QBankproAcademy.com

Explanation: The maroon-colored stool with a foul odor is indicative of lower gastrointestinal bleeding, and in this case, diverticulosis (B) is a likely source of the bleeding. Diverticulosis can lead to diverticular bleeding in the colon, resulting in dark, foul-smelling stools. Hemorrhoids (A) can cause bright red blood in the stool but typically do not produce maroon-colored stools. Gastric ulcers (C) and esophageal varices (D) are more likely to cause melena (dark, tarry stools) rather than maroon-colored stools.

digestion including gallbladder, liver, pancreas

NCLEX Question 69: Which symptom is most indicative of acute cholecystitis in a patient with gallstones?
A) Constipation
B) Hematuria
C) Right upper quadrant pain
D) Bilateral lower leg swelling

QBankproAcademy.com

Explanation: Acute cholecystitis is characterized by inflammation of the gallbladder, often due to gallstones blocking the cystic duct. The hallmark symptom is severe right upper quadrant pain, which typically radiates to the right shoulder or back. Options A, B, and D are not typical symptoms of acute cholecystitis.

NCLEX Question 70: What is the primary function of the gallbladder in the digestive process?
A) Production of digestive enzymes
B) Storage and concentration of bile
C) Absorption of nutrients
D) Regulation of blood glucose

QBankproAcademy.com

Explanation: The gallbladder stores and concentrates bile produced by the liver. When needed, it contracts to release bile into the small intestine to aid in the digestion and absorption of fats. Options A, C, and D are not primary functions of the gallbladder.

NCLEX Question 71: Which of the following dietary recommendations should be provided to a patient with gallstones to minimize the risk of gallbladder attacks?
A) High-fat diet
B) Low-fiber diet
C) Low-cholesterol diet
D) High-calcium diet

Explanation: Gallstones are often composed of cholesterol. Therefore, a low-cholesterol diet can help reduce the risk of gallstone formation and subsequent gallbladder attacks. Options A, B, and D are not recommended for individuals at risk of gallstones.

NCLEX Question 72: During a laparoscopic cholecystectomy, which precaution should the surgical team take to minimize the risk of common bile duct injury?
A) Ensure the patient remains in a supine position
B) Avoid using a laparoscope with a camera
C) Perform a cholangiogram to visualize the bile duct anatomy
D) Use a larger incision to improve visibility

QBankproAcademy.com

Explanation: A cholangiogram is a radiologic imaging technique that allows visualization of the bile duct anatomy during surgery, reducing the risk of common bile duct injury. Options A, B, and D are not recommended practices during a laparoscopic cholecystectomy and can increase the risk of complications.

NCLEX Question 73: Which risk factor is most strongly associated with the development of gallstones?
A) Regular exercise
B) Low body mass index (BMI)
C) Female gender
D) High dietary fiber intake

QBankproAcademy.com

Explanation: Gallstones are more common in women than men, with hormonal factors playing a significant role in their development. Estrogen, especially during pregnancy and the use of oral contraceptives, can increase the risk of gallstone formation. Options A, B, and D are not considered significant risk factors for gallstones.

NCLEX Question 74: What is the recommended dietary advice for a patient with a history of gallbladder removal (cholecystectomy)?
A) Low-fat diet
B) High-fiber diet
C) High-protein diet
D) Low-carbohydrate diet

QBankproAcademy.com

Explanation: After gallbladder removal, bile is continuously released into the small intestine, which can lead to difficulty digesting fats. Therefore, a low-fat diet is recommended to minimize gastrointestinal symptoms such as diarrhea and bloating. Options B, C, and D are not specific dietary recommendations for post-cholecystectomy patients and may not address their needs.

NCLEX Question 75: Which of the following is a primary function of the gallbladder?
A) Digestion of carbohydrates
B) Production of insulin
C) Storage and concentration of bile
D) Regulation of blood pressure

QBankproAcademy.com

Explanation: The primary function of the gallbladder is to store and concentrate bile produced by the liver. When needed, it releases bile into the small intestine to aid in the digestion and absorption of fats. Options A, B, and D are not functions of the gallbladder.

NCLEX Question 76: A patient with gallstones presents with severe right upper quadrant pain, nausea, and vomiting. What is the most likely diagnosis?
A) Cholecystitis
B) Pancreatitis
C) Appendicitis
D) Diverticulitis

QBankproAcademy.com

Explanation: The symptoms described, including severe right upper quadrant pain, nausea, and vomiting, are characteristic of acute cholecystitis, which is inflammation of the gallbladder, often caused by gallstones. Options B, C, and D describe conditions unrelated to the gallbladder.

NCLEX Question 77: A nurse is providing discharge instructions to a patient who has undergone a laparoscopic cholecystectomy. Which dietary recommendation should the nurse emphasize to the patient?
A) A high-fat diet
B) A low-fiber diet
C) A low-cholesterol diet
D) A high-calcium diet

QBankproAcademy.com

Explanation: After gallbladder removal, it is important to follow a low-cholesterol diet to reduce the risk of gallstone formation in the common bile duct. High-fat diets (option A) can be difficult to tolerate after cholecystectomy, and options B and D are not specific dietary recommendations for post-cholecystectomy patients.

NCLEX Question 78: ii8kiWhich laboratory finding is commonly associated with obstructive jaundice due to gallstones?
A) Elevated serum amylase levels
B) Decreased serum bilirubin levels
C) Elevated serum lipase levels
D) Elevated serum alkaline phosphatase (ALP) levels

Explanation: Obstructive jaundice caused by gallstones can lead to the obstruction of the common bile duct, resulting in an increase in serum ALP levels. Options A, B, and C are not typically associated with obstructive jaundice due to gallstones.

QBankproAcademy.com

Endocrine

NCLEX Question 79: Which hormone is primarily responsible for regulating blood sugar levels by promoting glucose uptake in cells?
A) Insulin
B) Glucagon
C) Somatostatin
D) Epinephrine

QBankproAcademy.com

Explanation: Insulin is the hormone produced by the pancreas, specifically by the beta cells in the islets of Langerhans, that helps regulate blood sugar levels by promoting the uptake of glucose into cells, thereby reducing blood glucose levels. Glucagon, on the other hand, raises blood sugar levels by stimulating the release of stored glucose from the liver.

NCLEX Question 80: What is the major function of the exocrine pancreas?
A) Regulation of blood sugar levels
B) Production of digestive enzymes
C) Production of insulin
D) Storage of bile

QBankproAcademy.com

Explanation: The exocrine pancreas is responsible for producing and secreting digestive enzymes into the small intestine to aid in the digestion and absorption of nutrients from the food we consume. It does not play a direct role in regulating blood sugar levels or storing bile.

NCLEX Question 81: Which condition is characterized by inflammation of the pancreas and is often associated with heavy alcohol consumption or gallstones?
A) Pancreatic cancer
B) Pancreatic cyst
C) Pancreatitis
D) Pancreatic abscess

QBankproAcademy.com

Explanation: Pancreatitis is an inflammatory condition of the pancreas that can be acute or chronic. It is frequently linked to alcohol abuse or gallstones. Pancreatic cancer, cysts, and abscesses are distinct conditions with different causes and characteristics.

NCLEX Question 82: In chronic pancreatitis, which vitamin deficiency is commonly observed due to impaired fat absorption?
A) Vitamin C deficiency
B) Vitamin A deficiency
C) Vitamin D deficiency
D) Vitamin B12 deficiency

QBankproAcademy.com

Explanation: Chronic pancreatitis can lead to impaired fat absorption, which, in turn, can result in a deficiency of fat-soluble vitamins such as vitamin D. This deficiency can lead to various health problems, including bone disorders. Other vitamin deficiencies may also occur, but vitamin D deficiency is a common consequence of fat malabsorption in chronic pancreatitis.

NCLEX Question 83: A 45-year-old female presents to the emergency department with severe right upper abdominal pain, nausea, and vomiting. She describes the pain as colicky and intermittent, worsening after consuming a fatty meal. Physical examination reveals tenderness in the right upper quadrant. Laboratory tests show elevated liver enzymes and a high white blood cell count. Which of the following conditions is the most likely diagnosis?
A) Cholecystitis
B) Hepatitis
C) Pancreatitis
D) Gastric ulcer

QBankproAcademy.com

Explanation: The patient’s clinical presentation of right upper quadrant pain, especially after fatty meals, along with tenderness and elevated liver enzymes, is suggestive of cholecystitis, which is inflammation of the gallbladder, often associated with gallstones.

NCLEX Question 84: A 50-year-old male complains of recurrent episodes of abdominal pain, particularly after meals, along with bloating and diarrhea. He has noticed that his stools appear pale in color and are floating. On further evaluation, a CT scan reveals dilation of the common bile duct. What is the most likely cause of these symptoms?

A) Gallstones
B) Cholecystitis
C) Pancreatic cancer
D) Alcoholic liver disease

QBankproAcademy.com

Explanation: The combination of abdominal pain, bloating, diarrhea, pale stools, and a dilated common bile duct suggests that gallstones have migrated into the common bile duct, causing obstruction and interfering with the normal flow of bile.

NCLEX Question 85: A 60-year-old female presents with jaundice, dark urine, and clay-colored stools. She reports experiencing unexplained weight loss over the past few months. On physical examination, the physician notes an enlarged, palpable gallbladder. Laboratory tests reveal elevated bilirubin levels. What is the most likely diagnosis?
A) Cholecystitis
B) Gallstones
C) Cholangiocarcinoma
D) Cirrhosis

QBankproAcademy.com

Explanation: The combination of jaundice, dark urine, clay-colored stools, unexplained weight loss, and an enlarged palpable gallbladder is highly concerning for cholangiocarcinoma, a malignant tumor of the bile ducts. This condition can obstruct the bile flow, leading to jaundice and other symptoms.

NCLEX Question 86: A 42-year-old female presents with severe right upper quadrant abdominal pain that started after she consumed a high-fat meal. The pain is steady and radiates to her right shoulder. She reports nausea and vomiting. Physical examination reveals Murphy’s sign, and laboratory tests show elevated liver enzymes and a high white blood cell count. What condition is most likely responsible for her symptoms?
A) Gallbladder polyp
B) Acute cholecystitis
C) Chronic pancreatitis
D) Appendicitis

QBankproAcademy.com

Explanation: The patient’s presentation with severe right upper quadrant pain, radiation to the right shoulder, Murphy’s sign (pain on palpation of the gallbladder area), and elevated liver enzymes is indicative of acute cholecystitis, which commonly occurs after the consumption of a fatty meal.

NCLEX Question 87: A 55-year-old male presents with recurrent episodes of indigestion and bloating, especially after meals. He also experiences belching and flatulence. He denies any significant abdominal pain. On examination, there is no tenderness or palpable masses. What is the most likely diagnosis for his gastrointestinal symptoms?
A) Gallbladder disease
B) Peptic ulcer disease
C) Gastroesophageal reflux disease (GERD)
D) Irritable bowel syndrome (IBS)

QBankproAcademy.com

Explanation: The patient’s recurrent indigestion, bloating, and symptoms exacerbated by meals may be indicative of gallbladder disease, such as gallstones or biliary dyskinesia. These conditions can lead to digestive discomfort without significant abdominal pain.

NCLEX Question 88: A 50-year-old female presents with jaundice, dark urine, and pale stools. She also complains of abdominal pain, which is more prominent in the epigastric region and radiates to the back. On examination, there is no palpable gallbladder, but an abdominal CT scan shows dilation of the common bile duct. What condition is most likely responsible for her symptoms?
A) Cholecystitis
B) Gallstones
C) Pancreatic cancer
D) Alcoholic liver disease

QBankproAcademy.com

Explanation: The combination of jaundice, dark urine, pale stools, abdominal pain radiating to the back, and common bile duct dilation is highly suggestive of pancreatic cancer, which can cause biliary obstruction and subsequent jaundice.

NCLEX Question 89: A 58-year-old male presents with right upper quadrant abdominal pain that occurs after consuming fatty meals. He has experienced multiple episodes of pain in the past, each lasting a few hours. He denies any recent fever or jaundice. Physical examination findings are unremarkable. What is the most likely diagnosis?
A) Gallbladder polyp
B) Acute cholecystitis
C) Gallstone ileus
D) Cholangitis

QBankproAcademy.com

Explanation: The patient’s recurrent right upper quadrant pain, especially after fatty meals, without signs of inflammation (fever or jaundice) may be indicative of a gallbladder polyp. Polyps in the gallbladder are typically benign and may cause intermittent pain.

NCLEX Question 90: A 35-year-old male presents with severe right upper abdominal pain that has been ongoing for several hours. The pain is constant and radiates to the back. He has a history of heavy alcohol use. On examination, he appears jaundiced, and Murphy’s sign is negative. Laboratory tests reveal elevated liver enzymes, bilirubin, and amylase levels. What condition is most likely responsible for his symptoms?
A) Gallstones
B) Alcoholic hepatitis
C) Acute pancreatitis
D) Peptic ulcer disease

QBankproAcademy.com

Explanation: This patient’s presentation with severe right upper abdominal pain, jaundice, elevated liver enzymes, and a history of heavy alcohol use is suggestive of acute pancreatitis. Alcohol consumption is a common cause of this condition, and it can present with symptoms similar to gallbladder-related issues.

NCLEX Question 91: A 45-year-old female presents with recurrent episodes of right upper quadrant pain, especially after consuming greasy meals. She also reports occasional bloating and flatulence. On examination, there is tenderness in the right upper quadrant. Ultrasound reveals the presence of gallstones. What is the most appropriate intervention for her condition?
A) Antibiotics
B) Cholecystectomy
C) Proton pump inhibitors (PPIs)
D) Antiemetics

QBankproAcademy.com

Explanation: In cases of symptomatic gallstones (cholelithiasis), the most appropriate treatment is cholecystectomy (removal of the gallbladder), especially when the patient experiences recurrent right upper quadrant pain and has confirmed gallstones on imaging.

NCLEX Question 92: A 55-year-old male presents with severe right upper abdominal pain, fever, and jaundice. He reports a history of gallstones and a recent episode of cholecystitis. On examination, there is right upper quadrant tenderness. Laboratory tests show elevated white blood cell count and liver enzymes. What complication of gallstones is likely occurring in this patient?
A) Gallstone ileus
B) Choledocholithiasis
C) Gallbladder polyp
D) Cholangitis

QBankproAcademy.com

Explanation: This patient’s history of gallstones, recent cholecystitis, right upper quadrant pain, fever, jaundice, and elevated white blood cell count are indicative of cholangitis, which is an infection and inflammation of the bile ducts. It is a serious complication of gallstones when they obstruct the common bile duct.

NCLEX Question 93: A 50-year-old female presents with mild, intermittent right upper abdominal discomfort. She denies any recent significant dietary changes or pain radiation. On examination, there is no tenderness or palpable masses. Laboratory tests are within normal limits. What diagnostic test is most appropriate to evaluate her symptoms?
A) Abdominal ultrasound
B) CT angiography
C) Upper endoscopy (esophagogastroduodenoscopy)
D) Colonoscopy

QBankproAcademy.com

Explanation: In patients with mild, intermittent right upper abdominal discomfort and no significant findings on physical examination or laboratory tests, an abdominal ultrasound is the most appropriate initial diagnostic test to evaluate for gallbladder-related issues, such as gallstones or gallbladder polyps

NCLEX Question 94: A 55-year-old male patient presents to the emergency department with jaundice, right upper quadrant abdominal pain, and dark urine. His medical history includes chronic alcohol abuse. On physical examination, the nurse observes hepatomegaly and spider angiomas. Laboratory results show elevated serum bilirubin, AST, ALT, and alkaline phosphatase levels. Which condition is most likely causing this patient’s symptoms?
A) Cirrhosis
B) Hepatitis A
C) Cholecystitis
D) Pancreatitis

QBankproAcademy.com

Explanation: This patient’s history of chronic alcohol abuse, jaundice, hepatomegaly, spider angiomas, and elevated liver enzyme levels (AST, ALT, alkaline phosphatase) are consistent with cirrhosis. Cirrhosis is characterized by chronic liver damage and scarring, often resulting from long-term alcohol abuse.

NCLEX Question 95: A 40-year-old female presents to the clinic with fatigue, abdominal swelling, and pedal edema. She reports a history of autoimmune hepatitis and is currently taking immunosuppressive medications. Physical examination reveals ascites and mild hepatomegaly. Laboratory tests show low albumin levels, elevated bilirubin, and prolonged prothrombin time. What is the most likely complication of her condition?
A) Hepatic encephalopathy
B) Hepatocellular carcinoma
C) Esophageal varices
D) Portal hypertension

QBankproAcademy.com

Explanation: The patient’s history of autoimmune hepatitis, ascites, hepatomegaly, low albumin levels, elevated bilirubin, and prolonged prothrombin time are indicative of cirrhosis, which can lead to portal hypertension. Portal hypertension is a common complication of cirrhosis and can result in the development of esophageal varices and other serious complications.

NCLEX Question 96: A 30-year-old male presents to the emergency department with severe right upper quadrant abdominal pain that radiates to the back. He has a history of gallstones and has experienced recurrent episodes of biliary colic. On examination, there is tenderness over the right upper quadrant, and Murphy’s sign is positive. Which complication is most likely associated with this presentation?
A) Cholecystitis
B) Hepatitis B
C) Acute pancreatitis
D) Peptic ulcer disease

QBankproAcademy.com

Explanation: The patient’s history of gallstones, right upper quadrant pain, and positive Murphy’s sign are consistent with acute cholecystitis, which is inflammation of the gallbladder often caused by gallstones. This condition can lead to severe abdominal pain and complications if not treated promptly.

NCLEX Question 97: A 65-year-old male with a history of nonalcoholic fatty liver disease (NAFLD) presents to the clinic for a routine follow-up. He has been working on improving his diet and exercise habits. Laboratory results show elevated liver enzymes, particularly ALT and AST, despite his efforts to lead a healthier lifestyle. What should the nurse advise this patient?
A) Continue with the current lifestyle changes, as they will eventually normalize liver enzymes.
B) Start a high-dose aspirin regimen to reduce inflammation.
C) Undergo a liver biopsy to assess the extent of liver damage.
D) Consult with a hepatologist to discuss further evaluation and management.

QBankproAcademy.com

Explanation: Elevated liver enzymes in a patient with a history of NAFLD may indicate ongoing liver inflammation or progression of the disease. Therefore, it is essential to consult with a hepatologist for further evaluation and management, which may include additional tests, medications, or interventions to prevent liver damage and complications.

NCLEX Question 98: A 45-year-old patient is admitted to the hospital with complaints of abdominal pain, nausea, and jaundice. Laboratory results indicate elevated serum bilirubin levels and increased alkaline phosphatase. The nurse suspects an obstruction in the biliary system. What diagnostic test should be performed to confirm the presence of a biliary obstruction?
A) Liver biopsy
B) Abdominal ultrasound
C) Colonoscopy
D) Cardiac stress test

QBankproAcademy.com

Explanation: An abdominal ultrasound is a non-invasive imaging test that can visualize the biliary system and detect any obstructions such as gallstones or tumors. It is a valuable diagnostic tool for identifying the cause of jaundice and abdominal pain in patients with liver or biliary issues.

NCLEX Question 99: A patient with advanced cirrhosis presents with altered mental status, asterixis (flapping tremor), and confusion. The nurse recognizes these as signs of which complication?
A) Hepatitis
B) Hepatocellular carcinoma
C) Hepatic encephalopathy
D) Cholecystitis

QBankproAcademy.com

Explanation: Hepatic encephalopathy is a neuropsychiatric complication of liver disease, particularly cirrhosis. It is characterized by altered mental status, asterixis (flapping tremor), confusion, and other neurological symptoms due to the accumulation of toxins in the bloodstream that the liver cannot efficiently clear.

NCLEX Question 100: A 30-year-old patient is diagnosed with viral hepatitis. The nurse educates the patient about modes of transmission. Which statement by the patient indicates a need for further education?
A) “I should avoid sharing needles or drug paraphernalia.”
B) “I can safely have unprotected sex as long as my partner also has hepatitis.”
C) “I should not share personal items like toothbrushes or razors.”
D) “I need to practice good hand hygiene to prevent the spread of the virus.”

QBankproAcademy.com

Explanation: The patient’s statement that unprotected sex is safe if both partners have hepatitis is incorrect. Hepatitis viruses can still be transmitted through sexual contact, and safe sex practices should be followed to prevent the spread of the virus.

NCLEX Question 101 A 55-year-old patient with liver cirrhosis is prescribed a diuretic to manage ascites. The nurse monitors the patient for potential complications. Which electrolyte imbalance is most concerning in this situation?
A) Hypokalemia
B) Hyponatremia
C) Hypercalcemia
D) Hypomagnesemia

Explanation: In a patient with liver cirrhosis and ascites, the use of diuretics can lead to hyponatremia (low sodium levels), which is a concerning electrolyte imbalance that can result in neurological symptoms such as confusion, seizures, and coma. Monitoring and managing sodium levels are crucial in such cases to prevent complications.

NCLEX Question 102: A 50-year-old patient is admitted to the hospital with a suspected liver disorder. The nurse observes that the patient’s skin has a yellowish hue, and scleral icterus is present. Which liver function test would most likely confirm the presence of liver dysfunction?
A) Serum creatinine level
B) Serum amylase level
C) Serum albumin level
D) Serum bilirubin level

Explanation: Elevated serum bilirubin levels are indicative of liver dysfunction and can result in jaundice, causing a yellowish hue to the skin and scleral icterus (yellowing of the whites of the eyes).

NCLEX Question 103: A patient with chronic liver disease is scheduled for a liver biopsy. The nurse should provide which pre-procedure instructions to the patient?A) “You must fast for 24 hours before the procedure.”
B) “Expect to stay in bed for 48 hours after the biopsy.”
C) “Avoid taking any prescribed medications on the morning of the biopsy.”
D) “Plan to lie on your right side after the procedure.”

Explanation: After a liver biopsy, the patient should lie on their right side for several hours to apply pressure to the biopsy site and minimize the risk of bleeding. Fasting, bed rest, and medication adjustments are not typically part of the pre-procedure instructions.

NCLEX Question 104: A patient with liver cirrhosis presents with massive ascites. The nurse understands that which medication is commonly used to manage ascites in this patient population?
A) Aspirin
B) Furosemide (Lasix)
C) Acetaminophen
D) Digoxin (Lanoxin)

Explanation: Furosemide is a loop diuretic commonly used to manage ascites in patients with liver cirrhosis. It helps reduce fluid retention by increasing urine output. Aspirin, acetaminophen, and digoxin are not typically used for this purpose.

NCLEX Question 105: A patient with liver disease is scheduled for a transjugular intrahepatic portosystemic shunt (TIPS) procedure. The nurse explains that the primary goal of TIPS is to:
A) Remove gallstones from the common bile duct.
B) Redirect blood flow in the hepatic artery.
C) Reduce portal hypertension and its complications.
D) Remove tumors from the liver.

Explanation: Transjugular intrahepatic portosystemic shunt (TIPS) is a procedure used to reduce portal hypertension, a common complication of liver disease. It creates a shunt to redirect blood flow and alleviate the increased pressure in the portal vein, which can help manage complications such as variceal bleeding and ascites.

NCLEX Question 106: A 40-year-old female patient presents to the emergency department with severe right upper quadrant abdominal pain, fever, and jaundice. Physical examination reveals tenderness over the gallbladder area, and laboratory results show elevated white blood cell count and bilirubin levels. Which condition is most likely causing this patient’s symptoms?
A) Cholecystitis
B) Pancreatitis
C) Appendicitis
D) Diverticulitis

Explanation: This patient’s symptoms, including right upper quadrant abdominal pain, fever, jaundice, and tenderness over the gallbladder area, are indicative of cholecystitis, which is inflammation of the gallbladder often caused by gallstones.

NCLEX Question 107: A 35-year-old patient has been experiencing recurrent episodes of biliary colic after consuming fatty meals. The nurse explains that the primary reason for these episodes is:

A) Gallbladder infection
B) Gallbladder obstruction
C) Gallbladder inflammation
D) Gallbladder enlargement

Explanation: Recurrent biliary colic is typically caused by gallbladder obstruction due to gallstones. These gallstones can intermittently block the flow of bile from the gallbladder, leading to painful episodes when the gallbladder contracts.

NCLEX Question 108: A 55-year-old male patient is scheduled for a laparoscopic cholecystectomy due to chronic gallbladder disease. The nurse explains that the primary reason for removing the gallbladder is:
A) To improve digestion of fats
B) To prevent the formation of new gallstones
C) To reduce the risk of pancreatic cancer
D) To relieve symptoms and prevent complications

Explanation: The primary reason for performing a cholecystectomy is to relieve symptoms and prevent complications associated with gallbladder disease, such as cholecystitis, biliary colic, and gallstone-related complications. The gallbladder is not essential for digestion, and its removal does not prevent the formation of new gallstones or reduce the risk of pancreatic cancer.

NCLEX Question 109: A patient is diagnosed with choledocholithiasis, which means there are gallstones in the common bile duct. The nurse explains that untreated choledocholithiasis can lead to which serious complication?
A) Peptic ulcer disease
B) Acute pancreatitis
C) Hypertension
D) Pneumonia

Explanation: Untreated choledocholithiasis, where gallstones obstruct the common bile duct, can lead to a backflow of bile into the pancreas, causing acute pancreatitis. This is a severe and potentially life-threatening complication of gallstone disease.

NCLEX Question 110: A 55-year-old male patient presents to the emergency department with jaundice, right upper quadrant abdominal pain, and dark urine. His medical history includes chronic alcohol abuse. On physical examination, the nurse observes hepatomegaly and spider angiomas. Laboratory results show elevated serum bilirubin, AST, ALT, and alkaline phosphatase levels. Which condition is most likely causing this patient’s symptoms?
A) Cirrhosis
B) Hepatitis A
C) Cholecystitis
D) Pancreatitis

Explanation: This patient’s history of chronic alcohol abuse, jaundice, hepatomegaly, spider angiomas, and elevated liver enzyme levels (AST, ALT, alkaline phosphatase) are consistent with cirrhosis. Cirrhosis is characterized by chronic liver damage and scarring, often resulting from long-term alcohol abuse.

NCLEX Question 111: Case: A 40-year-old female presents to the clinic with fatigue, abdominal swelling, and pedal edema. She reports a history of autoimmune hepatitis and is currently taking immunosuppressive medications. Physical examination reveals ascites and mild hepatomegaly. Laboratory tests show low albumin levels, elevated bilirubin, and prolonged prothrombin time. What is the most likely complication of her condition?
A) Hepatic encephalopathy
B) Hepatocellular carcinoma
C) Esophageal varices
D) Portal hypertension

Explanation: The patient’s history of autoimmune hepatitis, ascites, hepatomegaly, low albumin levels, elevated bilirubin, and prolonged prothrombin time are indicative of cirrhosis, which can lead to portal hypertension. Portal hypertension is a common complication of cirrhosis and can result in the development of esophageal varices and other serious complications.

NCLEX Question 112: A 30-year-old male presents to the emergency department with severe right upper quadrant abdominal pain that radiates to the back. He has a history of gallstones and has experienced recurrent episodes of biliary colic. On examination, there is tenderness over the right upper quadrant, and Murphy’s sign is positive. Which complication is most likely associated with this presentation?
A) Cholecystitis
B) Hepatitis B
C) Acute pancreatitis
D) Peptic ulcer disease

Explanation: The patient’s history of gallstones, right upper quadrant pain, and positive Murphy’s sign are consistent with acute cholecystitis, which is inflammation of the gallbladder often caused by gallstones. This condition can lead to severe abdominal pain and complications if not treated promptly.

NCLEX Question 113: A 65-year-old male with a history of nonalcoholic fatty liver disease (NAFLD) presents to the clinic for a routine follow-up. He has been working on improving his diet and exercise habits. Laboratory results show elevated liver enzymes, particularly ALT and AST, despite his efforts to lead a healthier lifestyle. What should the nurse advise this patient?
A) Continue with the current lifestyle changes, as they will eventually normalize liver enzymes.
B) Start a high-dose aspirin regimen to reduce inflammation.
C) Undergo a liver biopsy to assess the extent of liver damage.
D) Consult with a hepatologist to discuss further evaluation and management.

Explanation: Elevated liver enzymes in a patient with a history of NAFLD may indicate ongoing liver inflammation or progression of the disease. Therefore, it is essential to consult with a hepatologist for further evaluation and management, which may include additional tests, medications, or interventions to prevent liver damage and complications.

NCLEX Question 114: A 45-year-old patient is admitted to the hospital with complaints of abdominal pain, nausea, and jaundice. Laboratory results indicate elevated serum bilirubin levels and increased alkaline phosphatase. The nurse suspects an obstruction in the biliary system. What diagnostic test should be performed to confirm the presence of a biliary obstruction?
A) Liver biopsy
B) Abdominal ultrasound
C) Colonoscopy
D) Cardiac stress test

Explanation: An abdominal ultrasound is a non-invasive imaging test that can visualize the biliary system and detect any obstructions such as gallstones or tumors. It is a valuable diagnostic tool for identifying the cause of jaundice and abdominal pain in patients with liver or biliary issues.

NCLEX Question 115: A patient with advanced cirrhosis presents with altered mental status, asterixis (flapping tremor), and confusion. The nurse recognizes these as signs of which complication?
A) Hepatitis
B) Hepatocellular carcinoma
C) Hepatic encephalopathy
D) Cholecystitis

Explanation: Hepatic encephalopathy is a neuropsychiatric complication of liver disease, particularly cirrhosis. It is characterized by altered mental status, asterixis (flapping tremor), confusion, and other neurological symptoms due to the accumulation of toxins in the bloodstream that the liver cannot efficiently clear.

NCLEX Question 116: A 30-year-old patient is diagnosed with viral hepatitis. The nurse educates the patient about modes of transmission. Which statement by the patient indicates a need for further education?
A) “I should avoid sharing needles or drug paraphernalia.”
B) “I can safely have unprotected sex as long as my partner also has hepatitis.”
C) “I should not share personal items like toothbrushes or razors.”
D) “I need to practice good hand hygiene to prevent the spread of the virus.”

Explanation: The patient’s statement that unprotected sex is safe if both partners have hepatitis is incorrect. Hepatitis viruses can still be transmitted through sexual contact, and safe sex practices should be followed to prevent the spread of the virus.

NCLEX Question 117: A 55-year-old patient with liver cirrhosis is prescribed a diuretic to manage ascites. The nurse monitors the patient for potential complications. Which electrolyte imbalance is most concerning in this situation?
A) Hypokalemia
B) Hyponatremia
C) Hypercalcemia
D) Hypomagnesemia

Explanation: In a patient with liver cirrhosis and ascites, the use of diuretics can lead to hyponatremia (low sodium levels), which is a concerning electrolyte imbalance that can result in neurological symptoms such as confusion, seizures, and coma. Monitoring and managing sodium levels are crucial in such cases to prevent complications.

NCLEX Question 118: A 50-year-old patient is admitted to the hospital with a suspected liver disorder. The nurse observes that the patient’s skin has a yellowish hue, and scleral icterus is present. Which liver function test would most likely confirm the presence of liver dysfunction?
A) Serum creatinine level
B) Serum amylase level
C) Serum albumin level
D) Serum bilirubin level

Explanation: Elevated serum bilirubin levels are indicative of liver dysfunction and can result in jaundice, causing a yellowish hue to the skin and scleral icterus (yellowing of the whites of the eyes).

NCLEX Question 119: A patient with chronic liver disease is scheduled for a liver biopsy. The nurse should provide which pre-procedure instructions to the patient?
A) “You must fast for 24 hours before the procedure.”
B) “Expect to stay in bed for 48 hours after the biopsy.”
C) “Avoid taking any prescribed medications on the morning of the biopsy.”
D) “Plan to lie on your right side after the procedure.”

Explanation: After a liver biopsy, the patient should lie on their right side for several hours to apply pressure to the biopsy site and minimize the risk of bleeding. Fasting, bed rest, and medication adjustments are not typically part of the pre-procedure instructions.

NCLEX Question 120: A patient with liver cirrhosis presents with massive ascites. The nurse understands which medication is commonly used to manage ascites in this patient population?
A) Aspirin
B) Furosemide (Lasix)
C) Acetaminophen
D) Digoxin (Lanoxin)

Explanation: Furosemide is a loop diuretic commonly used to manage ascites in patients with liver cirrhosis. It helps reduce fluid retention by increasing urine output. Aspirin, acetaminophen, and digoxin are not typically used for this purpose.

NCLEX Question 121: A patient with liver disease is scheduled for a transjugular intrahepatic portosystemic shunt (TIPS) procedure. The nurse explains that the primary goal of TIPS is to:
A) Remove gallstones from the common bile duct.
B) Redirect blood flow in the hepatic artery.
C) Reduce portal hypertension and its complications.
D) Remove tumors from the liver.

Explanation: Transjugular intrahepatic portosystemic shunt (TIPS) is a procedure used to reduce portal hypertension, a common complication of liver disease. It creates a shunt to redirect blood flow and alleviate the increased pressure in the portal vein, which can help manage complications such as variceal bleeding and ascites.

NCLEX Question 122: A 40-year-old female patient presents to the emergency department with severe right upper quadrant abdominal pain, fever, and jaundice. Physical examination reveals tenderness over the gallbladder area, and laboratory results show elevated white blood cell count and bilirubin levels. Which condition is most likely causing this patient’s symptoms?
A) Cholecystitis
B) Pancreatitis
C) Appendicitis
D) Diverticulitis

Explanation: This patient’s symptoms, including right upper quadrant abdominal pain, fever, jaundice, and tenderness over the gallbladder area, are indicative of cholecystitis, which is inflammation of the gallbladder often caused by gallstones.

NCLEX Question 123: A 35-year-old patient has been experiencing recurrent episodes of biliary colic after consuming fatty meals. The nurse explains that the primary reason for these episodes is:
A) Gallbladder infection
B) Gallbladder obstruction
C) Gallbladder inflammation
D) Gallbladder enlargement

Explanation: Recurrent biliary colic is typically caused by gallbladder obstruction due to gallstones. These gallstones can intermittently block the flow of bile from the gallbladder, leading to painful episodes when the gallbladder contracts.

NCLEX Question 124: A 55-year-old male patient is scheduled for a laparoscopic cholecystectomy due to chronic gallbladder disease. The nurse explains that the primary reason for removing the gallbladder is:
A) To improve digestion of fats
B) To prevent the formation of new gallstones
C) To reduce the risk of pancreatic cancer
D) To relieve symptoms and prevent complications

Explanation: The primary reason for performing a cholecystectomy is to relieve symptoms and prevent complications associated with gallbladder disease, such as cholecystitis, biliary colic, and gallstone-related complications. The gallbladder is not essential for digestion, and its removal does not prevent the formation of new gallstones or reduce the risk of pancreatic cancer.

NCLEX Question 125: A patient is diagnosed with choledocholithiasis, which means there are gallstones in the common bile duct. The nurse explains that untreated choledocholithiasis can lead to which serious complication?
A) Peptic ulcer disease
B) Acute pancreatitis
C) Hypertension
D) Pneumonia

Explanation: Untreated choledocholithiasis, where gallstones obstruct the common bile duct, can lead to a backflow of bile into the pancreas, causing acute pancreatitis. This is a severe and potentially life-threatening complication of gallstone disease.

NCLEX Question 126: What is the term for the condition characterized by an enlarged spleen?
A) Splenomegaly
B) Splenitis
C) Splenectomy
D) Spleenitis

Explanation: Splenomegaly refers to the enlargement of the spleen, which can occur due to various underlying medical conditions such as infections, liver disease, or blood disorders.

NCLEX Question 127: In which abdominal quadrant is the spleen primarily located?
A) Right upper quadrant
B) Left upper quadrant
C) Right lower quadrant
D) Left lower quadrant

Explanation: The spleen is typically located in the left upper quadrant of the abdomen, just beneath the ribcage. This positioning allows it to filter blood and participate in the immune response.

NCLEX Question 128: Which of the following is a potential complication of a ruptured spleen?
A) Hypertension
B) Anemia
C) Osteoporosis
D) Diabetes

Explanation: A ruptured spleen can lead to internal bleeding, which can result in a significant loss of blood. Anemia is a common complication of internal bleeding, as it reduces the body’s ability to transport oxygen, leading to fatigue, weakness, and other symptoms.

NCLEX Question 129: A 32-year-old female presents to the emergency department with left-sided abdominal pain following a recent motor vehicle accident. On physical examination, she is found to have signs of shock. An abdominal CT scan reveals a ruptured spleen. What is the most appropriate initial intervention?
A) Administer antibiotics
B) Prepare for emergency splenectomy
C) Administer pain medication
D) Order a complete blood count (CBC)

Explanation: In the case of a ruptured spleen with signs of shock, the most appropriate initial intervention is emergency splenectomy to control the bleeding and prevent further deterioration in the patient’s condition.

NCLEX Question 130: A 45-year-old male with a history of cirrhosis presents with abdominal discomfort and early satiety. On physical examination, the healthcare provider notes splenomegaly. Which of the following is the most likely cause of the splenomegaly in this patient?
A) Viral infection
B) Alcohol consumption
C) Hypertension
D) Portal hypertension

Explanation: In patients with cirrhosis, portal hypertension can lead to the development of splenomegaly due to increased pressure in the portal vein system, causing blood to back up into the spleen.

NCLEX Question 131: A 28-year-old female is diagnosed with immune thrombocytopenic purpura (ITP), a condition characterized by a low platelet count. Which organ is primarily responsible for the destruction of platelets in ITP?
A) Liver
B) Spleen
C) Kidneys
D) Lungs

Explanation: In immune thrombocytopenic purpura (ITP), the spleen plays a key role in the destruction of platelets as it traps and removes antibody-coated platelets from circulation.

NCLEX Question 131: A 55-year-old male presents with fever, night sweats, and unintentional weight loss. Physical examination reveals splenomegaly. A complete blood count (CBC) shows leukocytosis with increased lymphocytes. What condition is most likely indicated by these findings?
A) Sickle cell anemia
B) Infectious mononucleosis
C) Chronic myeloid leukemia (CML)
D) Hemophilia

Explanation: Chronic myeloid leukemia (CML) is characterized by leukocytosis with an increased number of immature granulocytes and a significant increase in lymphocytes. Splenomegaly is a common finding in CML due to the accumulation of abnormal white blood cells in the spleen.

NCLEX Question 132: A 12-year-old boy presents to the pediatric clinic with severe pain in his joints and bones, along with fatigue. His medical history reveals recurrent hospitalizations for similar symptoms. On physical examination, the healthcare provider notices pallor and jaundice. A peripheral blood smear shows the presence of sickle-shaped red blood cells. What is the most appropriate initial nursing intervention for this patient?
A) Administer intravenous antibiotics
B) Encourage increased fluid intake
C) Prepare for a blood transfusion
D) Educate the patient on the importance of exercise

Explanation: The patient’s symptoms and the presence of sickle-shaped red blood cells suggest a vaso-occlusive crisis in sickle cell anemia. Increasing fluid intake is important to help prevent dehydration, which can exacerbate the crisis by making the blood more viscous and increasing the risk of further sickling of red blood cells.

NCLEX Question 133: A 28-year-old woman with sickle cell anemia presents to the emergency department with severe chest pain, cough, and difficulty breathing. She is tachypneic and has decreased oxygen saturation. What complication of sickle cell anemia is the patient likely experiencing?
A) Vaso-occlusive crisis
B) Acute chest syndrome
C) Aplastic crisis
D) Hemolytic crisis

Explanation: Acute chest syndrome is a life-threatening complication of sickle cell anemia characterized by chest pain, cough, and difficulty breathing. It often results from vaso-occlusion in the small blood vessels of the lungs and can lead to severe respiratory distress and decreased oxygen saturation. Immediate medical intervention, such as oxygen therapy and blood transfusion, is usually required to manage this condition.

Endocrine

NCLEX Question 134: A 45-year-old female presents with excessive thirst, frequent urination, and unexplained weight loss. Her blood glucose levels are consistently elevated. Which endocrine disorder is most likely responsible for her symptoms?
A) Hypothyroidism
B) Addison’s disease
C) Cushing’s syndrome
D) Diabetes mellitus

Explanation: The patient’s symptoms of excessive thirst, frequent urination, unexplained weight loss, and elevated blood glucose levels are characteristic of diabetes mellitus, a common endocrine disorder. Hypothyroidism (choice A) typically presents with fatigue, weight gain, and cold intolerance. Addison’s disease (choice B) presents with symptoms like fatigue, weakness, and low blood pressure. Cushing’s syndrome (choice C) is associated with weight gain, buffalo hump, and moon face.

NCLEX Question 135: A 30-year-old male complains of sudden, severe headaches, palpitations, excessive sweating, and anxiety. Upon evaluation, his blood pressure is significantly elevated. Which endocrine disorder is most likely responsible for his symptoms?
A) Hyperthyroidism
B) Pheochromocytoma
C) Hypoparathyroidism
D) Hypoglycemia

Explanation: The patient’s symptoms of severe headaches, palpitations, excessive sweating, and elevated blood pressure suggest pheochromocytoma, an adrenal gland tumor that releases excessive catecholamines. Hyperthyroidism (choice A) may cause anxiety and sweating but does not typically cause severe headaches and hypertension. Hypoparathyroidism (choice C) presents with muscle cramps and neuromuscular irritability, not the symptoms described. Hypoglycemia (choice D) may cause sweating and anxiety, but it is not typically associated with severe headaches and hypertension.

NCLEX Question 136: A 55-year-old female presents with fatigue, muscle weakness, and unintentional weight gain. Physical examination reveals a puffy face, thinning hair, and brittle nails. Laboratory results show elevated thyroid-stimulating hormone (TSH) levels and low free thyroxine (T4). What is the most likely diagnosis?
A) Hyperthyroidism
B) Hypoparathyroidism
C) Cushing’s syndrome
D) Hypothyroidism

Thinning hair, brittle nails, elevated TSH, and low free T4, are indicative of hypothyroidism. Hyperthyroidism (choice A) would present with opposite symptoms such as weight loss, anxiety, and increased heart rate. Hypoparathyroidism (choice B) typically manifests with neuromuscular irritability, not these symptoms. Cushing’s syndrome (choice C) leads to weight gain and characteristic features like buffalo hump and moon face, but it does not cause low thyroid hormone levels.

NCLEX Question 137: A 40-year-old male complains of persistent thirst, excessive urination, and visual disturbances. Laboratory results reveal hypernatremia and hyperglycemia. Which endocrine disorder is most likely responsible for his symptoms?
A) Diabetes mellitus
B) Hypoparathyroidism
C) Hyperthyroidism
D) Addison’s disease

Explanation: The patient’s symptoms of persistent thirst, excessive urination, hypernatremia (elevated sodium levels), and hyperglycemia are characteristic of diabetes mellitus. Hypoparathyroidism (choice B) typically presents with neuromuscular irritability, not these symptoms. Hyperthyroidism (choice C) may cause increased thirst and urination but is not associated with hypernatremia. Addison’s disease (choice D) typically presents with fatigue, weight loss, and low blood pressure, not the symptoms described.

NCLEX Question 138: A 23-year-old-female reports hair growth on her face and chest. On physical examination, you notice acanthosis nigricans. Which endocrine disorder is most likely responsible for her symptoms?
A) Cushing’s syndrome
B) Hyperthyroidism
C) Polycystic Ovary Syndrome (PCOS)
D) Addison’s disease

Explanation: The patient’s symptoms of irregular menstrual cycles, weight gain, excessive hair growth (hirsutism), and acanthosis nigricans are indicative of PCOS, an endocrine disorder associated with hormonal imbalances. Cushing’s syndrome (choice A) typically presents with different physical features and elevated cortisol levels. Hyperthyroidism (choice B) may cause irregular periods but is not typically associated with hirsutism. Addison’s disease (choice D) would present with fatigue and low blood pressure, among other symptoms, but not the symptoms described.

NCLEX Question 139: A 50-year-old male presents with extreme fatigue, muscle weakness, and unintentional weight loss. Laboratory results show elevated levels of serum calcium and low levels of parathyroid hormone (PTH). What is the most likely diagnosis?
A) Hyperparathyroidism
B) Hyperthyroidism
C) Hypoparathyroidism
D) Diabetes mellitus

Explanation: The patient’s symptoms of fatigue, muscle weakness, unintentional weight loss, elevated serum calcium, and low PTH levels are suggestive of hypoparathyroidism, an endocrine disorder characterized by inadequate parathyroid hormone production. Hyperparathyroidism (choice A) would lead to elevated PTH levels. Hyperthyroidism (choice B) typically presents with different symptoms such as weight loss, anxiety, and increased heart rate. Diabetes mellitus (choice D) is unrelated to the parathyroid gland and calcium regulation.

NCLEX Question 140: A 30-year-old male complains of excessive thirst, frequent urination, and blurred vision. His fasting blood glucose level is significantly elevated. Which endocrine disorder is most likely responsible for his symptoms?
A) Hypothyroidism
B) Addison’s disease
C) Diabetes mellitus
D) Acromegaly

Explanation: The patient’s symptoms of excessive thirst, frequent urination, elevated fasting blood glucose, and blurred vision are characteristic of diabetes mellitus, an endocrine disorder affecting blood sugar regulation. Hypothyroidism (choice A) typically presents with different symptoms like fatigue and weight gain. Addison’s disease (choice B) is associated with adrenal insufficiency and presents with fatigue and low blood pressure. Acromegaly (choice D) results from excess growth hormone and leads to enlarged features but is not related to the symptoms described.

NCLEX Question 141 A 25-year-old female presents with severe anxiety, tremors, palpitations, and weight loss despite increased appetite. On examination, you note an enlarged thyroid gland (goiter). Which endocrine disorder is most likely responsible for her symptoms?
A) Hypothyroidism
B) Pheochromocytoma
C) Hyperparathyroidism
D) Hyperthyroidism

Explanation: The patient’s symptoms of anxiety, tremors, palpitations, weight loss, increased appetite, and an enlarged thyroid gland (goiter) are characteristic of hyperthyroidism, an endocrine disorder resulting from excessive thyroid hormone production. Hypothyroidism (choice A) would present with opposite symptoms such as fatigue and weight gain. Pheochromocytoma (choice B) would typically manifest with severe headaches and hypertension, but it is not associated with thyroid enlargement. Hyperparathyroidism (choice C) primarily affects calcium regulation and would not explain the symptoms observed in this case.

NCLEX Question 142: A 40-year-old male presents with excessive thirst, frequent urination, and unexplained weight loss. Laboratory results show elevated fasting blood glucose levels. Which endocrine disorder is most likely responsible for his symptoms?
A) Hypothyroidism
B) Cushing’s syndrome
C) Diabetes mellitus
D) Addison’s disease

Explanation: The patient’s symptoms of excessive thirst, frequent urination, unexplained weight loss, and elevated fasting blood glucose levels are characteristic of diabetes mellitus, an endocrine disorder affecting blood sugar regulation. Hypothyroidism (choice A) typically presents with fatigue, weight gain, and cold intolerance. Cushing’s syndrome (choice B) is associated with weight gain, not weight loss. Addison’s disease (choice D) presents with fatigue, weight loss, and low blood pressure, but it does not cause elevated blood glucose levels.

NCLEX Question 143: A 30-year-old female presents with fatigue, muscle weakness, and weight gain. On examination, you notice puffiness around her eyes and a slowed heart rate. Which endocrine disorder is most likely responsible for her symptoms?
A) Hyperthyroidism
B) Hyperparathyroidism
C) Hypoparathyroidism
D) Hypothyroidism

Explanation: The patient’s symptoms of fatigue, muscle weakness, weight gain, puffiness around the eyes, and a slowed heart rate are indicative of hypothyroidism, an endocrine disorder resulting from inadequate thyroid hormone production. Hyperthyroidism (choice A) would present with opposite symptoms, such as weight loss and an increased heart rate. Hyperparathyroidism (choice B) primarily affects calcium regulation and would not explain these symptoms. Hypoparathyroidism (choice C) is associated with neuromuscular irritability and not typically with the symptoms described.

NCLEX Question 144: A 35-year-old male complains of sudden, severe headaches, palpitations, excessive sweating, and anxiety. On examination, his blood pressure is significantly elevated. Which endocrine disorder is most likely responsible for his symptoms?
A) Hypoparathyroidism
B) Pheochromocytoma
C) Hyperthyroidism
D) Diabetes mellitus

Explanation: The patient’s symptoms of severe headaches, palpitations, excessive sweating, anxiety, and significantly elevated blood pressure are suggestive of pheochromocytoma, an adrenal gland tumor that releases excessive catecholamines. Hypoparathyroidism (choice A) typically presents with neuromuscular irritability, not these symptoms. Hyperthyroidism (choice C) may cause palpitations and anxiety but is not typically associated with severe headaches and markedly elevated blood pressure. Diabetes mellitus (choice D) presents with different symptoms such as thirst and frequent urination.

NCLEX Question 145: A 50-year-old female presents with a hoarse voice, enlarged thyroid gland (goiter), and difficulty swallowing. Which endocrine disorder is most likely responsible for her symptoms?
A) Hypoparathyroidism
B) Hyperparathyroidism
C) Hashimoto’s thyroiditis
D) Cushing’s syndrome

Explanation: The patient’s symptoms of hoarse voice, enlarged thyroid gland (goiter), and difficulty swallowing are suggestive of Hashimoto’s thyroiditis, an autoimmune thyroid disorder leading to hypothyroidism and thyroid enlargement. Hypoparathyroidism (choice A) typically presents with neuromuscular irritability, not these symptoms. Hyperparathyroidism (choice B) primarily affects calcium regulation and would not explain the thyroid-related symptoms. Cushing’s syndrome (choice D) presents with weight gain and other features, but it is unrelated to the thyroid gland.

NCLEX Question 146: A 35-year-old female presents with excessive thirst, frequent urination, and unexplained weight loss. Laboratory results show elevated fasting blood glucose levels. Which endocrine disorder is most likely responsible for her symptoms?
A) Hypothyroidism
B) Cushing’s syndrome
C) Diabetes mellitus
D) Addison’s disease

Explanation: The patient’s symptoms of excessive thirst, frequent urination, unexplained weight loss, and elevated fasting blood glucose levels are characteristic of diabetes mellitus, an endocrine disorder affecting blood sugar regulation. Hypothyroidism (choice A) typically presents with fatigue, weight gain, and cold intolerance. Cushing’s syndrome (choice B) is associated with weight gain, not weight loss. Addison’s disease (choice D) presents with fatigue, weight loss, and low blood pressure, but it does not cause elevated blood glucose levels.

NCLEX Question 147: A 30-year-old female presents with fatigue, muscle weakness, and unintentional weight gain. On examination, you notice puffiness around her eyes and a slowed heart rate. Which endocrine disorder is most likely responsible for her symptoms?
A) Hyperthyroidism
B) Hyperparathyroidism
C) Hypoparathyroidism
D) Hypothyroidism

Explanation: The patient’s symptoms of fatigue, muscle weakness, weight gain, puffiness around the eyes, and a slowed heart rate are indicative of hypothyroidism, an endocrine disorder resulting from inadequate thyroid hormone production. Hyperthyroidism (choice A) would present with opposite symptoms, such as weight loss and an increased heart rate. Hyperparathyroidism (choice B) primarily affects calcium regulation and would not explain these symptoms. Hypoparathyroidism (choice C) is associated with neuromuscular irritability and not typically with the symptoms described.

nclex,hesi exit, ancc, aanp, questions and answers, adrenal gland disorders
The triangular-shaped adrenal gland

NCLEX Question 148: A 45-year-old male complains of sudden, severe headaches, palpitations, excessive sweating, and anxiety. On examination, his blood pressure is significantly elevated. Which endocrine disorder is most likely responsible for his symptoms?
A) Hypoparathyroidism
B) Pheochromocytoma
C) Hyperthyroidism
D) Diabetes mellitus

Explanation: The patient’s symptoms of severe headaches, palpitations, excessive sweating, anxiety, and significantly elevated blood pressure are suggestive of pheochromocytoma, an adrenal gland tumor that releases excessive catecholamines. Hypoparathyroidism (choice A) typically presents with neuromuscular irritability, not these symptoms. Hyperthyroidism (choice C) may cause palpitations and anxiety but is not typically associated with severe headaches and markedly elevated blood pressure. Diabetes mellitus (choice D) presents with different symptoms such as thirst and frequent urination.

NCLEX Question 149: A 50-year-old female presents with a hoarse voice, enlarged thyroid gland (goiter), and difficulty swallowing. Which endocrine disorder is most likely responsible for her symptoms?
A) Hypoparathyroidism
B) Hyperparathyroidism
C) Hashimoto’s thyroiditis
D) Cushing’s syndrome

Explanation: The patient’s symptoms of hoarse voice, enlarged thyroid gland (goiter), and difficulty swallowing are suggestive of Hashimoto’s thyroiditis, an autoimmune thyroid disorder leading to hypothyroidism and thyroid enlargement. Hypoparathyroidism (choice A) typically presents with neuromuscular irritability, not these symptoms. Hyperparathyroidism (choice B) primarily affects calcium regulation and would not explain the thyroid-related symptoms. Cushing’s syndrome (choice D) presents with weight gain and other features, but it is unrelated to the thyroid gland.

NCLEX Question 150: A 35-year-old female presents with excessive thirst, frequent urination, and unexplained weight loss. Laboratory results show elevated fasting blood glucose levels. Which endocrine disorder is most likely responsible for her symptoms?
A) Hypothyroidism
B) Cushing’s syndrome
C) Diabetes mellitus
D) Addison’s disease

Explanation: The patient’s symptoms of excessive thirst, frequent urination, unexplained weight loss, and elevated fasting blood glucose levels are characteristic of diabetes mellitus, an endocrine disorder affecting blood sugar regulation. Hypothyroidism (choice A) typically presents with fatigue, weight gain, and cold intolerance. Cushing’s syndrome (choice B) is associated with weight gain, not weight loss. Addison’s disease (choice D) presents with fatigue, weight loss, and low blood pressure, but it does not cause elevated blood glucose levels.

AANP, ANCC, NCLEX, questions and answers HESI Exit, thyroid anatomy, thyroid disorders, endocrine
Thyroid anatomy

NCLEX Question 151: A 30-year-old female presents with fatigue, muscle weakness, and unintentional weight gain. On examination, you notice puffiness around her eyes and a slowed heart rate. Which endocrine disorder is most likely responsible for her symptoms?
A) Hyperthyroidism
B) Hyperparathyroidism
C) Hypoparathyroidism
D) Hypothyroidism

Explanation: The patient’s symptoms of fatigue, muscle weakness, weight gain, puffiness around the eyes, and a slowed heart rate are indicative of hypothyroidism, an endocrine disorder resulting from inadequate thyroid hormone production. Hyperthyroidism (choice A) would present with opposite symptoms, such as weight loss and an increased heart rate. Hyperparathyroidism (choice B) primarily affects calcium regulation and would not explain these symptoms. Hypoparathyroidism (choice C) is associated with neuromuscular irritability and not typically with the symptoms described.

NCLEX Question 151: A 45-year-old male complains of sudden, severe headaches, palpitations, excessive sweating, and anxiety. On examination, his blood pressure is significantly elevated. Which endocrine disorder is most likely responsible for his symptoms?
A) Hypoparathyroidism
B) Pheochromocytoma
C) Hyperthyroidism
D) Diabetes mellitus

Explanation: The patient’s symptoms of severe headaches, palpitations, excessive sweating, anxiety, and significantly elevated blood pressure are suggestive of pheochromocytoma, an adrenal gland tumor that releases excessive catecholamines. Hypoparathyroidism (choice A) typically presents with neuromuscular irritability, not these symptoms. Hyperthyroidism (choice C) may cause palpitations and anxiety but is not typically associated with severe headaches and markedly elevated blood pressure. Diabetes mellitus (choice D) presents with different symptoms such as thirst and frequent urination.

Adrenal Tumor, nclex, aanp, ancc, guestions and answers, qbank
Adrenal Tumor

NCLEX Question 152: A 50-year-old female presents with a hoarse voice, enlarged thyroid gland (goiter), and difficulty swallowing. Which endocrine disorder is most likely responsible for her symptoms?
A) Hypoparathyroidism
B) Hyperparathyroidism
C) Hashimoto’s thyroiditis
D) Cushing’s syndrome

Explanation: The patient’s symptoms of hoarse voice, enlarged thyroid gland (goiter), and difficulty swallowing are suggestive of Hashimoto’s thyroiditis, an autoimmune thyroid disorder leading to hypothyroidism and thyroid enlargement. Hypoparathyroidism (choice A) typically presents with neuromuscular irritability, not these symptoms. Hyperparathyroidism (choice B) primarily affects calcium regulation and would not explain the thyroid-related symptoms. Cushing’s syndrome (choice D) presents with weight gain and other features, but it is unrelated to the thyroid gland.

Renal

NCLEX Question 153: A 65-year-old male with a history of hypertension presents to the emergency department with complaints of decreased urine output, swelling in his lower extremities, and fatigue. Physical examination reveals elevated blood pressure and pitting edema in the legs. Laboratory results show increased serum creatinine and decreased glomerular filtration rate (GFR). What is the most likely diagnosis?
A) Acute pyelonephritis
B) Chronic kidney disease (CKD)
C) Acute renal failure
D) Renal artery stenosis

Explanation: The patient’s history of hypertension, decreased GFR, elevated serum creatinine, and clinical manifestations of fluid retention and fatigue are consistent with chronic kidney disease (CKD). CKD is characterized by gradual loss of kidney function over time and is often asymptomatic until it reaches advanced stages.

NCLEX Question 154: A 28-year-old female presents with severe flank pain, hematuria, and nausea. She has a family history of kidney stones. A non-contrast computed tomography (CT) scan of the abdomen reveals a 5 mm stone in the left ureter causing obstruction. What is the most appropriate initial management for this patient?
A) Surgical removal of the stone
B) Intravenous antibiotics
C) Increased fluid intake and pain control
D) Administration of corticosteroids

Explanation: The patient’s symptoms and imaging findings are consistent with a ureteral stone causing obstruction. The initial management for uncomplicated ureteral stones is increased fluid intake to promote stone passage and pain control. Surgical intervention may be considered if conservative management fails or if there are complications.

NCLEX Question 155: A 50-year-old male with a history of diabetes mellitus and hypertension presents with complaints of polyuria, polydipsia, and unintentional weight loss over the past few months. Laboratory tests show hyperglycemia, ketonuria, and an elevated serum creatinine level. What is the most likely diagnosis?
A) Diabetic nephropathy
B) Diabetic ketoacidosis (DKA)
C) Hypertensive nephrosclerosis
D) Acute tubular necrosis (ATN)

Explanation: The patient’s symptoms, including polyuria, polydipsia, hyperglycemia, ketonuria, and elevated serum creatinine, are consistent with diabetic ketoacidosis (DKA). DKA is a severe complication of uncontrolled diabetes characterized by hyperglycemia, metabolic acidosis, and electrolyte imbalances.

NCLEX Question 156: A 72-year-old female with a history of heart failure and chronic kidney disease (CKD) presents with shortness of breath, peripheral edema, and fatigue. Physical examination reveals jugular venous distention and crackles on lung auscultation. Laboratory tests show elevated serum B-type natriuretic peptide (BNP) levels. What is the most likely cause of her symptoms?
A) Acute kidney injury (AKI)
B) Pulmonary embolism
C) Congestive heart failure exacerbation
D) Urinary tract infection (UTI)

Explanation: The patient’s history of heart failure, CKD, shortness of breath, peripheral edema, elevated BNP levels, and physical examination findings are consistent with a congestive heart failure exacerbation. In patients with CKD, fluid overload can exacerbate heart failure symptoms, leading to pulmonary congestion and peripheral edema.

NCLEX Question 157: A 45-year-old male presents to the clinic with complaints of frequent urination, thirst, and fatigue. Laboratory tests reveal elevated blood glucose levels and glucose in the urine. Which of the following conditions is most likely responsible for these symptoms?
A) Polycystic kidney disease
B) Diabetic nephropathy
C) Chronic kidney disease (CKD)
D) Urinary tract infection (UTI)

Explanation: The symptoms of frequent urination, thirst, fatigue, elevated blood glucose levels, and glucose in the urine are indicative of diabetes mellitus. Diabetic nephropathy is a common complication of diabetes that affects the kidneys, leading to proteinuria and progressive kidney damage.

Renal Failure Nursing Diagnosis
A patient with renal failure receiving hemodialysis

NCLEX Question 158: A nurse is caring for a patient with end-stage renal disease (ESRD) who is on hemodialysis. Which electrolyte imbalance is commonly associated with ESRD and requires close monitoring during hemodialysis?
A) Hypercalcemia
B) Hyperkalemia
C) Hyponatremia
D) Hypophosphatemia

Explanation: Patients with end-stage renal disease (ESRD) often have impaired potassium excretion, leading to hyperkalemia. During hemodialysis, the removal of excess potassium is essential to prevent life-threatening cardiac arrhythmias.

NCLEX Question 159: A 30-year-old female with a history of urinary tract infections (UTIs) presents with fever, flank pain, and dysuria. A urinalysis reveals pyuria and bacteriuria. Which of the following conditions is the most likely cause of her symptoms?
A) Nephrotic syndrome
B) Acute glomerulonephritis
C) Renal calculi (kidney stones)
D) Acute pyelonephritis

Explanation: The patient’s symptoms of fever, flank pain, dysuria, pyuria, and bacteriuria are indicative of acute pyelonephritis, which is a severe kidney infection often associated with a history of recurrent UTIs.

NCLEX Question 160: A 55-year-old male with a history of hypertension is found to have a narrowing of the renal arteries, resulting in reduced blood flow to the kidneys. This condition is most commonly referred to as:
A) Renal cell carcinoma
B) Renal artery stenosis
C) Glomerulonephritis
D) Nephrotic syndrome

Explanation: Renal artery stenosis is the narrowing of one or both renal arteries, typically due to atherosclerosis, and can result in decreased blood flow to the kidneys. This condition can lead to hypertension and impaired kidney function.

NCLEX Question 161: A 55-year-old female with a history of chronic kidney disease (CKD) presents with severe fatigue, weakness, and nausea. Laboratory tests reveal a hemoglobin level of 7.5 g/dL. Which of the following is the most likely cause of her anemia?
A) Iron deficiency anemia
B) Hemolytic anemia
C) Chronic disease anemia
D) Aplastic anemia

Explanation: Anemia is a common complication of chronic kidney disease (CKD). It is typically characterized as a “chronic disease anemia” due to factors such as decreased production of erythropoietin and impaired iron utilization. This leads to a reduced number of red blood cells, resulting in fatigue and weakness.

NCLEX Question 162: A 68-year-old male with a history of hypertension presents with periorbital edema, foamy urine, and hypoalbuminemia. Which of the following conditions is the most likely diagnosis?
A) Nephrotic syndrome
B) Acute glomerulonephritis
C) Renal calculi (kidney stones)
D) Urinary tract infection (UTI)

Explanation: The patient’s symptoms of periorbital edema, foamy urine (indicative of proteinuria), and hypoalbuminemia are characteristic of nephrotic syndrome. Nephrotic syndrome is a kidney disorder in which there is excessive protein loss in the urine, leading to edema and low serum albumin levels.

NCLEX Question 163: A 42-year-old female with a history of diabetes mellitus presents with complaints of burning sensation during urination, increased urinary frequency, and cloudy urine. Urinalysis reveals leukocyte esterase and nitrite positivity. What is the most likely diagnosis?
A) Chronic kidney disease (CKD)
B) Diabetic nephropathy
C) Urinary tract infection (UTI)
D) Acute glomerulonephritis

Explanation: The patient’s symptoms and urinalysis findings are consistent with a urinary tract infection (UTI). UTIs can cause symptoms such as dysuria, increased urinary frequency, and cloudy urine and are more common in individuals with diabetes.

NCLEX Question 164: A 60-year-old male presents with acute onset of severe flank pain that radiates to the groin. He is diaphoretic and restless. A non-contrast computed tomography (CT) scan of the abdomen reveals a 4 mm stone in the right ureter. What is the most appropriate initial management for this patient?
A) Intravenous antibiotics
B) Surgical removal of the stone
C) Increased fluid intake and pain control
D) Administration of corticosteroids

Explanation: The patient’s symptoms and imaging findings are consistent with a ureteral stone causing obstruction. The initial management for uncomplicated ureteral stones is increased fluid intake to promote stone passage and pain control. Surgical intervention may be considered if conservative management fails or if there are complications.

NCLEX Question 165: A 60-year-old male with a history of hypertension and diabetes mellitus presents to the emergency department with complaints of lower back pain, frequent urination, and hematuria. Physical examination reveals hypertension. Urinalysis shows the presence of red blood cells and protein. Which condition is most likely responsible for his symptoms?
A) Kidney stones (renal calculi)
B) Urinary tract infection (UTI)
C) Acute glomerulonephritis
D) Polycystic kidney disease

Explanation: The patient’s symptoms of lower back pain, hematuria, and hypertension are indicative of kidney stones (renal calculi). Hematuria and flank pain are common clinical manifestations of kidney stones, which can occur in individuals with risk factors like hypertension and diabetes.

NCLEX Question 166: A 45-year-old female is admitted to the hospital with complaints of extreme thirst, polyuria, and fatigue. Laboratory tests reveal hyperglycemia, glucosuria, and ketonuria. What is the most likely diagnosis?
A) Nephrotic syndrome
B) Diabetic nephropathy
C) Diabetic ketoacidosis (DKA)
D) Chronic kidney disease (CKD)

Explanation: The patient’s symptoms of extreme thirst, polyuria, hyperglycemia, glucosuria, and ketonuria are classic signs of diabetic ketoacidosis (DKA), a severe complication of uncontrolled diabetes mellitus.

NCLEX Question 167: A 50-year-old male with a history of hypertension and tobacco use presents with sudden-onset severe abdominal pain radiating to the flank, gross hematuria, and nausea. Physical examination reveals hypertension and costovertebral angle tenderness. Which of the following is the most likely diagnosis?
A) Kidney stones (renal calculi)
B) Appendicitis
C) Diverticulitis
D) Cholecystitis

Explanation: The patient’s symptoms of sudden-onset severe abdominal pain radiating to the flank, gross hematuria, hypertension, and costovertebral angle tenderness are indicative of kidney stones (renal calculi). These symptoms commonly occur when a stone obstructs the urinary tract.

NCLEX Question 168: A 70-year-old female with a history of congestive heart failure presents with bilateral lower extremity edema, elevated serum creatinine, and decreased urine output. Which of the following conditions is the most likely cause of her renal impairment?
A) Acute tubular necrosis (ATN)
B) Acute glomerulonephritis
C) Diabetic nephropathy
D) Nephrotic syndrome

Explanation: The patient’s history of congestive heart failure, lower extremity edema, elevated serum creatinine, and decreased urine output are suggestive of acute tubular necrosis (ATN). ATN is often seen in patients with reduced renal perfusion, such as those with congestive heart failure, and can lead to acute kidney injury.

NCLEX Question 169: A 40-year-old male with a history of type 2 diabetes mellitus and hypertension presents with complaints of swelling in the ankles, fatigue, and decreased urine output. Physical examination reveals elevated blood pressure and pitting edema in the legs. Laboratory results show increased serum creatinine and decreased glomerular filtration rate (GFR). Which condition is most likely responsible for his symptoms?
A) Acute pyelonephritis
B) Chronic kidney disease (CKD)
C) Acute renal failure
D) Renal artery stenosis

Explanation: The patient’s history of diabetes and hypertension, along with symptoms of edema, elevated blood pressure, increased serum creatinine, and decreased GFR, are consistent with chronic kidney disease (CKD). CKD is characterized by gradual loss of kidney function over time and often presents with symptoms in advanced stages.

NCLEX Question 170: A 30-year-old female presents with severe flank pain, hematuria, and nausea. She has a family history of kidney stones. A non-contrast computed tomography (CT) scan of the abdomen reveals a 5 mm stone in the left ureter causing obstruction. What is the most appropriate initial management for this patient?
A) Surgical removal of the stone
B) Intravenous antibiotics
C) Increased fluid intake and pain control
D) Administration of corticosteroids

Explanation: The initial management for uncomplicated ureteral stones causing obstruction is increased fluid intake to promote stone passage and pain control. Surgical intervention may be considered if conservative management fails or if there are complications.

NCLEX Question 171: A 55-year-old male with a history of hypertension is found to have a narrowing of the renal arteries, resulting in reduced blood flow to the kidneys. This condition is most commonly referred to as:
A) Renal cell carcinoma
B) Renal artery stenosis
C) Glomerulonephritis
D) Nephrotic syndrome

Explanation: Renal artery stenosis is the narrowing of one or both renal arteries, often due to atherosclerosis, which can lead to reduced blood flow to the kidneys. This condition is associated with hypertension and impaired kidney function.

NCLEX Question 172: A 25-year-old female presents with fever, flank pain, and costovertebral angle tenderness. A urinalysis shows leukocyte esterase and nitrite positivity, and a urine culture grows Escherichia coli. What is the most likely diagnosis?
A) Nephrotic syndrome
B) Acute glomerulonephritis
C) Renal calculi (kidney stones)
D) Acute pyelonephritis

Explanation: The patient’s symptoms of fever, flank pain, costovertebral angle tenderness, leukocyte esterase and nitrite positivity in urinalysis, and the presence of Escherichia coli in the urine culture are consistent with acute pyelonephritis, a bacterial infection of the kidney.

Musculoskeletal

NCLEX Question 173: Which assessment finding indicates a potential complication of a fractured long bone in a patient immobilized for an extended period?
A) Increased muscle strength
B) Decreased joint range of motion
C) Increased peripheral pulses
D) Decreased skin temperature

Explanation: Prolonged immobilization can lead to reduced blood flow and tissue perfusion, resulting in decreased skin temperature over the affected area. This is often seen as a potential complication in patients with fractures and immobility.

NCLEX Question 174: A nurse is caring for a patient with osteoarthritis. Which intervention is most appropriate for managing the patient’s pain?
A) Administering nonsteroidal anti-inflammatory drugs (NSAIDs)
B) Applying ice packs to the affected joints
C) Encouraging high-impact aerobic exercise
D) Advising the patient to avoid weight-bearing activities

Explanation: NSAIDs are commonly used to manage pain and inflammation associated with osteoarthritis. They help reduce pain and improve joint function by reducing inflammation.

NCLEX Question 175: A patient with a hip fracture is scheduled for surgery. Which action should the nurse prioritize before the surgical procedure?
A) Administering a laxative to prevent constipation
B) Assisting the patient with ambulation in the hallway
C) Providing pain medication as requested
D) Keeping the affected hip immobilized

Explanation: Prior to hip surgery, the nurse should ensure that the affected hip remains immobilized to prevent further injury and complications. Ambulation and laxatives can be addressed post-surgery as appropriate.

NCLEX Question 176: A patient is receiving a cast for a wrist fracture. Which statement by the patient indicates a need for further education?
A) “I should avoid sticking objects down the cast to scratch an itch.”
B) “I can use a hairdryer on the cool setting to speed up the drying process.”
C) “It’s normal to have some swelling and discomfort after the cast is applied.”
D) “I will keep my hand elevated to reduce swelling and promote healing.”

Explanation: Using a hairdryer, even on the cool setting, can cause uneven drying of the cast material and potentially lead to skin burns or discomfort. Patients should be advised against this practice and informed about appropriate cast care measures.

NCLEX Question 177: A 62-year-old male patient presents to the emergency department with severe lower back pain after lifting a heavy object at work. He reports a sudden onset of pain that radiates down his left leg. Upon assessment, the nurse notes decreased range of motion in the lumbar spine and tenderness over the lower back area. The patient also complains of numbness and tingling in his left leg. Which musculoskeletal condition is most likely responsible for the patient’s symptoms?
A) Plantar fasciitis
B) Rotator cuff tear
C) Lumbar herniated disc
D) Carpal tunnel syndrome

Explanation: The patient’s sudden onset of severe lower back pain, radiation down the left leg, numbness, and tingling are indicative of a lumbar herniated disc, also known as a “slipped” or “ruptured” disc.

NCLEX Question 178: What physical examination finding should the nurse expect when assessing the patient’s lower back?
A) Increased lumbar spine flexibility
B) Normal muscle strength in the lower extremities
C) Decreased sensation in the affected leg
D) Limited lumbar spine range of motion

Explanation: Patients with a lumbar herniated disc often experience limited range of motion in the lumbar spine due to pain and muscle guarding.

NCLEX Question 179: Which intervention is a priority for managing the patient’s acute pain and preventing further exacerbation of symptoms?
A) Administering a heating pad to the lower back
B) Encouraging the patient to perform lumbar stretches
C) Administering analgesic medication as prescribed
D) Instructing the patient to lift heavy objects with a straight back

Explanation: Administering analgesic medication is a priority to manage the patient’s acute pain. Heat or stretching exercises should be considered after pain is controlled, and proper lifting techniques should be discussed as part of education to prevent future injuries.

NCLEX Question 180: The nurse should educate the patient about the importance of which activity to minimize further episodes of back pain?
A) Heavy weightlifting
B) Avoiding all physical activity
C) Maintaining good posture
D) Sitting for prolonged periods

Explanation: Maintaining good posture is essential for minimizing the risk of further back pain episodes. It helps distribute the load evenly on the spine and reduces the strain on the lumbar discs, promoting musculoskeletal health.

NCLEX Question 181: A 45-year-old female patient arrives at the orthopedic clinic with a chief complaint of knee pain. She reports that the pain began gradually and has been worsening over the past few months. She mentions that it is particularly painful when she climbs stairs or stands up after sitting for an extended period. On examination, the nurse notes crepitus in the knee joint and tenderness over the patellar region. Which musculoskeletal condition is most likely responsible for the patient’s symptoms?
A) Plantar fasciitis
B) Achilles tendonitis
C) Osteoarthritis of the knee
D) Carpal tunnel syndrome

Explanation: The patient’s age, gradual onset of knee pain, pain with specific activities (stairs, prolonged sitting), crepitus, and tenderness over the patellar region are suggestive of osteoarthritis of the knee.

NCLEX Question 182 What radiographic finding is commonly associated with osteoarthritis?
A) Increased joint space
B) Normal bone density
C) Erosion of joint surfaces
D) Thickening of joint cartilage

Explanation: Osteoarthritis often presents with radiographic findings such as the erosion or loss of joint surface cartilage, joint space narrowing, and the formation of bone spurs (osteophytes).

NCLEX Question 183: Which non-pharmacological intervention should the nurse recommend to help manage the patient’s knee pain?
A) Application of cold packs to the knee
B) Administering over-the-counter NSAIDs
C) Weight-bearing exercises
D) Physical therapy and joint-strengthening exercises

Explanation: Physical therapy and joint-strengthening exercises are important components of non-pharmacological management for osteoarthritis. They help improve joint stability and function.

NCLEX Question 184:  What patient education point is essential for the nurse to emphasize regarding osteoarthritis?
A) The importance of high-impact aerobic exercise
B) The use of corticosteroid injections for pain relief
C) The need for prolonged bed rest during symptom exacerbation
D) The benefits of maintaining a healthy body weight

Explanation: Maintaining a healthy body weight is crucial for patients with osteoarthritis as excess weight places additional stress on weight-bearing joints, exacerbating symptoms. Weight management is an essential aspect of osteoarthritis management.

NCLEX Question 185: A 68-year-old male patient presents to the primary care clinic with complaints of joint pain and stiffness, primarily affecting his hands and wrists. He mentions that the pain is most severe in the morning and gradually improves as the day progresses. The patient has a history of smoking and reports occasional shortness of breath. On examination, the nurse observes swelling of the proximal interphalangeal (PIP) joints, as well as skin thickening on the fingers.

Based on the patient’s presentation, which musculoskeletal condition is most likely responsible for his symptoms?
A) Osteoarthritis
B) Rheumatoid arthritis
C) Gout
D) Ankylosing spondylitis

Explanation: The patient’s symptoms of joint pain and stiffness, especially in the morning, along with swelling of the PIP joints and skin thickening, are indicative of rheumatoid arthritis.

NCLEX Question 186:  Which laboratory test is commonly used to aid in the diagnosis of rheumatoid arthritis?
A) Serum uric acid level
B) C-reactive protein (CRP)
C) Erythrocyte sedimentation rate (ESR)
D) Serum creatinine level

Explanation: CRP is a marker of inflammation and is often elevated in individuals with rheumatoid arthritis. It is used as a diagnostic tool to assess disease activity and monitor treatment response.

NCLEX Question 187:  In addition to pharmacological treatments, which lifestyle modification should the nurse recommend to manage the patient’s rheumatoid arthritis symptoms?
A) Smoking cessation
B) Increasing dietary purine intake
C) High-impact aerobic exercise
D) Prolonged bed rest

Explanation: Smoking is a known risk factor for the development and exacerbation of rheumatoid arthritis. Smoking cessation is an important lifestyle modification to help manage the condition.

NCLEX Question 188:  The nurse should educate the patient about the potential long-term effects of untreated rheumatoid arthritis, which may include:

A) Decreased risk of cardiovascular disease
B) Improved lung function
C) Joint deformities and disability
D) Increased bone density

Explanation: Untreated rheumatoid arthritis can lead to joint deformities and disability over time due to progressive joint damage and inflammation. Early intervention and treatment are essential to minimize these complications.

NCLEX Question 189:  A 30-year-old female presents to the emergency department with severe pain and limited range of motion in her right shoulder. She reports that the pain began suddenly after she tried to lift a heavy suitcase while on vacation. On examination, the nurse observes that the patient is holding her right arm close to her body and is unable to actively abduct or externally rotate her shoulder. There is tenderness over the top of her shoulder, and the nurse notes a visible deformity in the shoulder joint. What is the most likely diagnosis based on the patient’s presentation and physical examination findings?

A) Rotator cuff tear
B) Tennis elbow (lateral epicondylitis)
C) Carpal tunnel syndrome
D) Achilles tendonitis

Explanation: The sudden onset of severe shoulder pain, limited range of motion, tenderness, and a visible deformity in the shoulder joint are indicative of a rotator cuff tear, which commonly occurs with lifting or trauma.

NCLEX Question 191:  Which diagnostic test is typically used to confirm the diagnosis of a rotator cuff tear?
A) X-ray of the shoulder
B) Magnetic resonance imaging (MRI)
C) Bone scan
D) Ultrasound of the elbow

Explanation: MRI is a valuable diagnostic tool for visualizing soft tissues, including the rotator cuff tendons and any tears or abnormalities. It is commonly used to confirm the diagnosis of a rotator cuff tear.

Musculoskeletal NCLEX, QUESTIONS, muscles, joints, bones, nclex, aanp, ancc, guestions and answers, qbank
Shoulder Joint

NCLEX Question 192:  What initial intervention should the nurse prioritize for the management of the patient’s suspected rotator cuff tear?
A) Administering corticosteroid injections
B) Immobilizing the shoulder in a sling
C) Initiating physical therapy and range of motion exercises
D) Applying heat packs to the shoulder

Explanation: Immobilization with a sling is an initial intervention to protect the shoulder and prevent further damage in the case of a suspected rotator cuff tear. It helps minimize movement and strain on the affected area.

NCLEX Question 193:  The nurse should educate the patient about the importance of which factor in the recovery process for a rotator cuff tear?
A) Heavy lifting and strenuous exercise
B) Avoiding any shoulder movement
C) Compliance with prescribed physical therapy
D) Applying cold packs to the shoulder

Explanation: Physical therapy plays a crucial role in the rehabilitation and recovery of a rotator cuff tear. It helps improve shoulder strength and range of motion and is essential for achieving the best possible outcome.

NCLEX Question 194:A patient with a history of gallstones presents with severe right upper quadrant pain, fever, and jaundice. Which condition is most likely indicated by these symptoms?
A) Chronic cholecystitis
B) Gallbladder polyps
C) Choledocholithiasis
D) Biliary dyskinesia

Explanation: Choledocholithiasis refers to the presence of gallstones in the common bile duct. Symptoms typically include severe right upper quadrant pain, fever, and jaundice due to obstruction of the bile flow. Options A, B, and D are less likely to present with these specific symptoms.

NCLEX Question 195: Which function is primarily associated with the spleen?
A) Red blood cell production
B) Digestion of fats
C) Immune response
D) Regulation of blood glucose

Explanation: The spleen plays a crucial role in the immune system by filtering and removing damaged or old red blood cells and platelets, as well as participating in the immune response by producing antibodies and storing immune cells

Mental Health

NCLEX Question 196: A 32-year-old female, has been diagnosed with major depressive disorder and is prescribed an antidepressant. During her follow-up appointment, she mentions experiencing increased restlessness, agitation, and insomnia since starting the medication. What should the nurse suspect?
A) She is developing a tolerance to the medication.
B) She is experiencing a paradoxical reaction.
C) Her depression is worsening.
D) She is experiencing normal side effects of the medication.

Explanation: A paradoxical reaction is when a patient responds to a medication in an unexpected manner, experiencing symptoms opposite to the intended therapeutic effect. In this case, the patient has restlessness, agitation, and insomnia may be indicative of a paradoxical reaction to the antidepressant.

DEPRESSION, nclex, aanp, ancc, guestions and answers, qbank
Symptoms of Depression

NCLEX Question 197: Mark, a 45-year-old male, has been admitted to the psychiatric unit due to auditory hallucinations and disorganized thinking. The nurse observes Mark talking to himself and responding to internal stimuli that are not present. Which mental health condition is most likely affecting Mark?
A) Generalized Anxiety Disorder (GAD)
B) Bipolar Disorder
C) Schizophrenia
D) Obsessive-Compulsive Disorder (OCD)

Explanation: Mark’s symptoms, including auditory hallucinations and disorganized thinking, are characteristic of schizophrenia, a severe and chronic mental health disorder that often involves hallucinations, delusions, and impaired cognition.

Schizophrenia, ECG, EKG, nclex, aanp, ancc, guestions and answers, qbank
Schizophrenia, hallucinations

NCLEX Question 198: Maria, a 33-year-old nurse, has been working long hours at a busy hospital during the COVID-19 pandemic. She feels physically and emotionally drained, has trouble sleeping, and experiences a persistent sense of hopelessness. What assessment finding is consistent with Maria’s symptoms?
A) Hypomania
B) Burnout
C) Panic Disorder
D) Borderline Personality Disorder

Explanation: Maria’s symptoms of feeling physically and emotionally drained, trouble sleeping, and a persistent sense of hopelessness are indicative of burnout, which is often seen in individuals who experience chronic stress and overwork.

NCLEX Question 199: John, a 22-year-old college student, has been missing classes, social activities, and has withdrawn from friends and family. He reports feeling worthless and having difficulty concentrating. What is the primary nursing intervention for John?
A) Administer an antipsychotic medication.
B) Refer John for psychoanalysis therapy.
C) Perform a suicide risk assessment.
D) Encourage John to attend group therapy.

Explanation: Given John’s symptoms of social withdrawal, feelings of worthlessness, and difficulty concentrating, it is crucial to perform a suicide risk assessment to determine if he poses a risk to himself. Assessing suicide risk is a priority in the care of individuals with depression.

NCLEX QUESTIONS, Depression, Mental Health
Depression

NCLEX Question 200: Alex, a 19-year-old individual, has been exhibiting impulsive and risky behaviors, such as reckless driving, excessive spending, and promiscuity. He also report feeling euphoric and needing very little sleep. What mental health condition should the nurse suspect?
A) Major Depressive Disorder (MDD)
B) Borderline Personality Disorder (BPD)
C) Bipolar Disorder (BD)
D) Generalized Anxiety Disorder (GAD)

Explanation: Alex’s symptoms of impulsivity, risk-taking, euphoria, and decreased need for sleep are indicative of a manic episode, a characteristic feature of Bipolar Disorder.

Bipolar Disorder, nclex, aanp, ancc, guestions and answers, qbank
Bipolar Disorder

NCLEX Question 201: Sarah, a 35-year-old woman, experiences intense fear and anxiety when in crowded places or enclosed spaces. She avoids public transportation and crowded events due to her fear. What is the most likely diagnosis for Sarah?
A) Social Anxiety Disorder
B) Agoraphobia
C) Panic Disorder
D) Obsessive-Compulsive Disorder (OCD)

Explanation: Sarah’s fear of crowded places and avoidance of certain situations are classic signs of agoraphobia, which is an anxiety disorder often associated with panic attacks and avoidance behavior.

NCLEX Question 202: A 50-year-old man, has been experiencing persistent, intrusive thoughts, even though he has no intention of acting on these thoughts. He feels distressed and ashamed of these thoughts. What mental health condition is James likely dealing with? A) Obsessive-Compulsive Disorder (OCD) B) Major Depressive Disorder (MDD) C) Generalized Anxiety Disorder (GAD) D) Post-Traumatic Stress Disorder (PTSD)

Explanation: James’s experience of distressing, intrusive thoughts is a common symptom of Obsessive-Compulsive Disorder, often referred to as “obsessions.”

NCLEX Question 203: A 25-year-old college student, has been feeling extremely sad and hopeless for the past several weeks. She has lost interest in activities she once enjoyed and is experiencing significant changes in her appetite and sleep patterns. What condition should the nurse consider as a possible diagnosis?
A) Bipolar Disorder
B) Borderline Personality Disorder (BPD)
C) Major Depressive Disorder (MDD)
D) Schizophrenia

Explanation: The patient’s symptoms of persistent sadness, hopelessness, loss of interest, changes in appetite and sleep, are indicative of Major Depressive Disorder, a mood disorder characterized by a pervasive low mood and loss of interest or pleasure in most activities.

NCLEX Question 204: Sarah, a 40-year-old woman, has been experiencing intense fear and avoidance of leaving her home for the past six months. She is unable to attend social events, go to work, or even grocery shopping. What is the most likely diagnosis for Sarah?
A) Social Anxiety Disorder
B) Generalized Anxiety Disorder
C) Agoraphobia
D) Panic Disorder

Explanation: Sarah’s intense fear of leaving her home and avoiding various situations is characteristic of agoraphobia, an anxiety disorder characterized by avoidance of places or situations from which escape may be difficult or embarrassing.

NCLEX Question 205: Mark, a 18-year-old man, frequently experiences episodes of intense fear and discomfort, along with symptoms such as a racing heart, shortness of breath, chest pain, and a feeling of impending doom. These episodes occur suddenly and unexpectedly. What is the most likely diagnosis for Mark?
A) Social Anxiety Disorder
B) Panic Disorder
C) Generalized Anxiety Disorder
D) Obsessive-Compulsive Disorder

Explanation: Mark’s recurrent episodes of intense fear and physical symptoms, such as racing heart and chest pain, are indicative of panic attacks, a hallmark of Panic Disorder.

Panic attack, nclex, aanp, ancc, guestions and answers, qbank
Panic attack

NCLEX Question 206: A 25-year-old college student, has been experiencing excessive worry and anxiety about her academic performance and future career. She finds it difficult to control her worry and often experiences physical symptoms such as restlessness and muscle tension. What condition is Emily likely dealing with?
A) Post-Traumatic Stress Disorder (PTSD)
B) Major Depressive Disorder (MDD)
C) Generalized Anxiety Disorder (GAD)
D) Bipolar Disorder

Explanation: The patient’s excessive worry, difficulty in controlling it, and accompanying physical symptoms like restlessness and muscle tension are characteristic features of Generalized Anxiety Disorder (GAD), a chronic condition marked by excessive and uncontrollable worry.

NCLEX Question 207: John, a 60-year-old man, has been experiencing auditory hallucinations, disorganized thinking, and delusions of grandeur. He believes he is a world-famous musician and often talks to voices only he can hear. What mental health condition is most likely affecting John?
A) Schizophrenia
B) Bipolar Disorder
C) Major Depressive Disorder (MDD)
D) Generalized Anxiety Disorder (GAD)

Explanation: John’s symptoms of auditory hallucinations, disorganized thinking, and delusions of grandeur are characteristic of schizophrenia, a severe and chronic mental disorder characterized by disturbances in thought, perception, and behavior.

Need more MED SURG Questions? Medical Surgical Practice Tests: 1 2 3 4 5 6 7 8 9 10